2 закон ньютона задачи с решением: Тюменский индустриальный университет » Страница не найдена

Содержание

Урок физики «Решение задач по теме “Законы Ньютона”»

Цель урока: Закрепление и применение знаний по теме “Законы Ньютона” при решении качественных и количественных задач.

Задачи урока :

Образовательные: отработать навыки применения законов Ньютона.

Развивающие:

  • развивать монологическую речь учащихся, умение ставить вопросы и отвечать на них;
  • развивать умение учащихся работать с графиками.

Воспитательные:

  • воспитать любознательность, внимательность, усидчивость;
  • воспитать трудолюбие, точность и четкость при ответе, умение видеть физику вокруг себя;
  • ориентировать учащихся на использование теоретических знаний в жизни и практической деятельности.

ХОД УРОКА

1) Организационный момент.

Здравствуйте! Садитесь!

2) Подготовка к активной учебно-познавательной деятельности.

Сегодня на уроке мы проверим домашнее задание, повторим законы Ньютона, а затем перейдем к решению задач по теме “Законы Ньютона”.

3) Проверка знаний полученных на прошлых занятиях.

Фронтальный опрос по вопросам:

  • 1 закон Ньютона
  • 2 закон Ньютона
  • 3 закон ньютона

4) Закрепление ранее полученных знаний, решение качественных и количественных задач.

1. Как объяснить, что бегущий человек, споткнувшись, падает в направлении своего движения, а поскользнувшись, падает в направлении, противоположном направлению своего движения?

Решение: Это явление легко объясняется на основании первого закона Ньютона. Бегущий человек. Споткнувшись, падает в направлении своего движения. Потому что при этом ноги человека замедляют движение. А туловище сохраняет по инерции прежнее состояние движения. В то время как ноги начинают скользить вперед быстрее, потому человек падает назад.

2.Парашютист падает с постоянной по модулю скоростью. Чему равен модуль силы сопротивления воздуха при этом движении?

Решение: Движение парашютиста равномерное и прямолинейное, поэтому, на основании 1 закона Ньютона, все силы, действующие на парашютиста, компенсируются. Так как парашютист движется под действием силы тяжести, то сила сопротивления воздуха по модулю равна силе тяжести парашютиста и противоположно направлена.

3. Как направленно ускорение самолета, если на него действует 4 силы: по вертикали – сила тяжести = 200кН и подъемная сила 210кН. По горизонтали: сила тяжести мотора 20 кН и сила лобового сопротивления воздуха 10 кН. Чему равна равнодействующая всех сил?

Дано:

F1 = 2*10 5 H

F2 = 2, 1*10 5 H

F3 = 2*10 4 H

F4 =10 4 H

R-?

Решение:

R1 = F2 – F1, (по вертикали)

R1 = 2, 1*105 – 2*105 = 104Н

R1 = F3 – F4, (по горизонтали)

R1 = 2*10 4 – 10 4 =10 4 H

Найдем равнодействующую всех сил, пользуясь правилом параллелограмма:

= +

Модуль силы R вычислим с помощью теоремы Пифагора: R=

R = =1, 4*10 4 Н

Ответ: Равнодействующая всех сил направлена под углом под углом 45°к горизонту и равна 1,4*10 4 Н.

4. Под действием силы в 20 Н материальная точка движется с а=0,4 м/с2. С каким ускорением будет двигаться точка под действием силы в 50 Н?

Дано:

F1 = 20 H

a1 = 0, 4 м/с

F2 = 50 H

Решение:

Движение материальной точки является равноускоренным.

По второму закону Ньютона можно определить массу материальной точки.

m=

m=

=

=

Ответ: а2 = 1 м/с

5. К пристани причаливают две одинаковые лодки. Лодочники подтягиваются к берегу с помощью веревок. Противоположный конец первой веревки привязан к столбу на пристани; за противоположный конец второй веревки тянет матрос, стоящий на пристани. Все трое прилагают одинаковые усилия. Какая лодка причалит раньше?

Решение: Обе лодки причалят одновременно. По третьему закону Ньютона каждому действию есть равное противодействие. С какой силой лодочники тянут за один конец веревки, с такой же силой второй конец веревки действует на столб и на матроса. Другими словами, столб “тянет” конец веревки с такой же силой, с какой ее тянет матрос, стоящий на пристани.

6. На тело массой 2160 кг, лежащее на горизонтальной дороге, действует сила, под действием которой тело за 30 секунд пройдет расстояние 500 метров. Найти величину этой силы.

Дано:

m=2160кг

t=30c

S=500м

F-?

Решение:

F=ma

;

Ответ: 2400 Н

5) Контроль знаний:

Самостоятельная работа по вариантам:

  • 1 вариант: третий закон Ньютона (формулировка, вывод), равнодействующая сил, задача № 152(Рымкевич)
  • 2 вариант: второй закон Ньютона (формулировка, вывод), первый закон Ньютона с новой формулировкой, задача № 156(Рымкевич)

6) Домашнее задание.

  • Выучить формулы.

Литература.

  1. А.В. Перышкин, Е. М. Гутник “Физика. 9 класс”.
  2. Сборник задач по физике. П.А.Рымкевич.

Второй закон Ньютона. Связь силы, массы и ускорения тела

Второй закон Ньютона в изначальной формулировке автора гласил:

«Изменение количества движения пропорционально приложенной силе и происходит в направлении силы».

Это главный закон Ньютона, на котором основаны практически все расчеты в инерционной механике и который определяет взаимосвязь между массой тела, его ускорением и силами, действующими на это тело.

Связь силы, массы и ускорения тела

Как следует из закона инерции, ускорение материальной точки равно нулю, если точка изолирована от действия сил. Если же к точке приложена некоторая сила, то эта точка отклоняется от инерционного движения, приобретая некоторое ускорение.

Опыт показывает, что одна и та же сила сообщает двум различным покоящимся материальным точкам неодинаковые ускорения.

При действии одинаковых сил модуль ускорения одной материальной точки может оказаться меньшим, чем модуль ускорения другой точки. В этом случае говорят, что первая точка является более инертной, и наоборот. Отсюда следует, что

инертность выражает свойство материальной точки под действием силы приобретать в своем движении то или иное ускорение.

Таким образом, ускорения, приобретаемые двумя различными материальными точками, зависят не только от действующих на них сил, но и от инертности этих точек.

Величина, определяющая меру инертности данной материальной точки, называется массой точки. Масса является величиной скалярной, положительной, и при этом в классической механике она рассматривается как величина постоянная для каждой данной материальной точки.

Масса является единственной характеристикой материальной точки. Отличие материальных точек друг от друга сводится к различию в массах.

Связь между массой m материальной точки, силой F, приложенной к этой точке, и сообщаемым ею ускорением a устанавливается вторым законом динамики. Приведем этот закон в следующей формулировке:

Ускорение материальной точки относительно инерциальной системы отсчета пропорционально действующей на точку силе, направлено по этой силе и обратно пропорционально массе точки, т.е.

Следует заметить, что из формулировки второго закона динамики не вытекает, что в динамике исследуются движения, происходящие только в инерциальных системах отсчета. В динамике рассматривается движение и в неинерциальных системах, но только таких, движение которых задано относительно инерциальной системы отсчета.

Приведем здесь еще одну формулировку второго закона Ньютона, в современной трактовке:

Равнодействующая всех сил, приложенных к телу, равна произведению массы этого тела на его ускорение, под действием этих сил.

Третий закон Ньютона >

Порядок проведения вступительного испытания по физике (ВШТЭ)

На все направления подготовки в Высшей школе технологии и энергетики экзамен проводится очно с личным присутствием в аудитории, в случае, если это не противоречит постановлениям губернатора Санкт-Петербурга, издаваемым в соответствии с Указом Президента РФ от 11 мая 2020 г. №316 «Об определении порядка продления действия мер по обеспечению санитарно-эпидемиологического благополучия населения в субъектах Российской Федерации в связи с распространением новой коронавирусной инфекции (СOVID-19)», исходя из санитарно-эпидемиологической обстановки и особенностей распространения новой коронавирусной инфекции (СOVID-19).

Вступительный экзамен по физике проводится в виде письменной работы.

1. Продолжительность экзамена – 3 часа (180 минут).
2. Работа выполняется синей или черной пастой. Записи, сделанные карандашом, не проверяются.
3.Разрешается использовать непрограммируемый калькулятор.
4. Все необходимые для решения справочные данные и значения физических постоянных приводятся в условиях задач.
5. Решение задач желательно сопровождать краткими пояснениями.
6. Ответы выписываются дважды: после решения каждой задачи и в сводной таблице, приводимой на четвертой странице чистого двойного листа.
7. Каждое задание содержит 12 задач. №1-6 – задачи-тесты, к каждой из них дается несколько вариантов ответа, из которых верен только один. Каждая задача оценивается в 5 баллов. №7-10 – задачи первого класса сложности. Каждая задача оценивается в 10 баллов. №11, 12 – задачи второго класса сложности, одна задача оценивается в 15 баллов.
8. Максимальная сумма баллов при полном правильном решении равна 100 баллам (6х5+4х10+2х15=100)
9. При проведении экзамена с применением дистанционных технологий абитуриент не позднее чем через 10 минут после завершения задания должен загрузить на платформу фотографии черновиков с решениями заданий. Без черновиков работа не проверяется.

 

Механика

Кинематика. Механическое движение. Относительность движения.

Система отсчета. Материальная точка. Траектория. Путь и перемещение.

Скорость. Ускорение.

Равномерное и равноускоренное прямолинейное движение. Свободное падение тел. Ускорение свободного падения. Уравнение прямолинейного равноускоренного движения.

Криволинейное движение точки на примере движения по окружности с постоянной по модулю скоростью.

Центростремительное ускорение.

Основы динамики. Инерция. Первый закон Ньютона. Инерциальные системы отсчета.

Взаимодействие тел. Масса. Импульс. Сила. Второй закон Ньютона.

Принцип суперпозиции сил. Принцип относительности Галилея.

Силы в природе. Сила тяготения. Закон всемирного тяготения. Вес тела. Невесомость. Первая космическая скорость. Сила упругости. Закон

Гука. Сила трения. Коэффициент трения. Закон трения скольжения.

Третий закон Ньютона.

Момент силы. Условие равновесия тел.

Законы сохранения в механике. Закон сохранения импульса. Ракеты.

Механическая работа. Мощность. Кинетическая энергия.

Потенциальная энергия. Закон сохранения энергии в механике. Простые механизмы. Коэффициент полезного действия механизма.

Механика жидкостей и газов. Давление. Атмосферное давление.

Изменение атмосферного давления с высотой. Закон Паскаля для жидкостей и газов. Барометры и манометры. Сообщающиеся сосуды. Принцип устройства гидравлического пресса.

Архимедова сила для жидкостей и газов. Условия плавания тел на поверхности жидкости.

Движение жидкости по трубам. Зависимость давления жидкости от скорости ее течения.

Измерение расстояний, промежутков времени, силы, объема, массы, атмосферного давления.

Молекулярная физика. Термодинамика

Основы молекулярно-кинетической теории. Опытное обоснование основных положений молекулярно-кинетической теории. Броуновское движение. Диффузия. Масса и размер молекул. Измерение скорости молекул. Опыт Штерна. Количество вещества. Моль. Постоянная Авогадро.

Взаимодействие молекул. Модели газа, жидкости и твердого тела.

Основы термодинамики. Тепловое равновесие. Температура и ее измерение. Абсолютная температурная шкала. Внутренняя энергия.

Количество теплоты. Теплоемкость вещества. Работа в термодинамике.

Первый закон термодинамики. Изотермический, изохорный и изобарный процессы. Адиабатный процесс.

Необратимость тепловых процессов. Второй закон термодинамики и его статистическое истолкование. Преобразование энергии в тепловых двигателях. КПД теплового двигателя.

Идеальный газ. Связь между давлением и средней кинетической энергией молекул идеального газа. Связь температуры со средней кинетической энергией частиц газа.

Уравнение Клапейрона-Менделеева. Универсальная газовая постоянная.

Жидкости и твердые тела. Испарение и конденсация. Насыщенные и ненасыщенные пары. Влажность воздуха. Кипение жидкости.

Кристаллические и аморфные тела. Преобразование энергии при изменениях агрегатного состояния вещества.

Измерение давления газа, влажности воздуха, температуры, плотности вещества.

Основы электродинамики

Электростатика. Электризация тел. Электрический заряд.

Взаимодействие зарядов. Элементарный электрический заряд. Закон сохранения электрического заряда. Закон Кулона.

Электрическое поле. Напряженность электрического поля.

Электрическое поле точечного заряда. Потенциальность электростатического поля. Разность потенциалов. Принцип суперпозиции полей.

Проводники в электрическом поле. Электрическая емкость.

Конденсатор. Емкость плоского конденсатора.

Диэлектрики в электрическом поле. Диэлектрическая проницаемость.

Энергия электрического поля плоского конденсатора.

Постоянный электрический ток. Электрический ток. Сила тока.

Напряжение. Носители свободных электрический зарядов в металлах, жидкостях и газах. Сопротивление проводников. Закон Ома для участка цепи. Последовательное и параллельное соединение проводников.

Электродвижущая сила. Закон Ома для полной цепи. Работа и мощность тока. Закон Джоуля-Ленца.

Полупроводники. Собственная и примесная проводимость полупроводников, p-n-переход.

Магнитное поле, электромагнитная индукция. Взаимодействие магнитов. Взаимодействие проводников с током. Магнитное поле. Действие магнитного поля на электрические заряды. Индукция магнитного поля.

Сила Ампера. Сила Лоренца. Магнитный поток. Электродвигатель.

Электромагнитная индукция. Закон электромагнитной индукции

Фарадея. Правило Ленца. Вихревое электрическое поле. Самоиндукция.

Индуктивность. Энергия магнитного поля.

Измерение силы тока, напряжения, сопротивления проводника.

Колебания и волны

Механические колебания и волны. Гармонические колебания.

Амплитуда, период и частота колебаний. Свободные колебания.

Математический маятник. Период колебаний математического маятника.

Превращение энергии при гармонических колебаниях. Вынужденные колебания. Резонанс. Понятие об автоколебаниях.

Механические волны. Скорость распространения волны. Длина волны.

Поперечные и продольные волны. Уравнение гармонической волны.

Звук.

Электромагнитные колебания и волны. Колебательный контур.

Свободные электромагнитные колебания в контуре. Превращение энергии в колебательном контуре. Собственная частота колебаний в контуре.

Вынужденные электрические колебания. Переменный электрический ток.

Генератор переменного тока. Действующие значения силы тока и напряжения. Активное, емкостное и индуктивное сопротивления. Резонанс в электрической цепи.

Трансформатор. Производство, передача и потребление электрической энергии.

Идеи теории Максвелла. Электромагнитные волны. Скорость распространения электромагнитных волн. Свойства электромагнитных волн.

Принципы радиосвязи. Шкала электромагнитных волн.

Оптика

Свет – электромагнитная волна. Прямолинейное распространение, отражение и преломление света. Луч. Законы отражения и преломления света. Показатель преломления. Полное отражение. Предельный угол полного отражения. Ход лучей в призме. Построение изображений в плоском зеркале.

Собирающая и рассеивающая линзы. Формула тонкой линзы.

Построение изображений в линзах. Фотоаппарат. Глаз. Очки.

Интерференция света. Когерентность. Дифракция света. Дифракционная решетка. Поляризация света. Поперечность световых волн.

Дисперсия света.

Измерение фокусного расстояния собирающей линзы, показателя преломления вещества, длины волны света.

Основы специальной теории относительности

Инвариантность скорости света. Принцип относительности Эйнштейна.

Пространство и время в специальной теории относительности. Связь массы и энергии.

Квантовая физика

Тепловое излучение. Постоянная Планка. Фотоэффект. Опыты

Столетова. Уравнение Эйнштейна для фотоэффекта.

Гипотеза Луи де Бройля. Дифракция электронов. Корпускулярно- волновой дуализм.

Радиоактивность. Альфа-, бета-, гамма-излучения. Методы наблюдения и регистрации частиц в ядерной физике.

Опыт Резерфорда по рассеянию альфа-частиц. Планетарная модель атома. Боровская модель атома водорода. Спектры. Люминесценция.

Лазеры.

Закон радиоактивного распада. Нуклонная модель ядра. Заряд ядра.

Массовое число ядра. Энергия связи частиц в ядре. Деление ядер. Синтез ядер. Ядерные реакции. Сохранение заряда и массового числа при ядерных реакциях. Выделение энергии при делении и синтезе ядер. Использование ядерной энергии. Дозиметрия.

Элементарные частицы. Фундаментальные взаимодействия.

Методы научного познания и физическая картина мира

Эксперимент и теория в процессе познания мира. Моделирование явлений и объектов природы. Научные гипотезы. Физические законы и границы их применимости. Роль математики в физике. Принцип соответствия. Принцип причинности. Физическая картина мира.

Урок физики на тему «Законы Ньютона». – Учительская газета

Образовательные задачи: проверить усвоение обучающимися основных теоретических понятий, физических формул данной темы и внести коррекцию, выявить уровень сформированности умений обучающихся решать задачи, пользоваться табличными данными, провести предварительную диагностику достижений.

Впы:       Д       В фч ч, пч       С, ы ж , , ых       С фч ч,       Пб ы       С, пп ж п пх       В фч ч, ы       Ецы ы   ы б ч ф   Т : «Д Зы » Г ч – бб , пчы п ч ы «Зы »   А э бх п ы б   Уч бх п ы, пы ч, ч пы   Впы:       Дп ф п       К пы ?       Д х п пы ?       Дп ф       Мч п       Дп ф       С ч ы ? ( пц, ы 4-9)   П бы ч пч п п «Зы », ж пч п ш б (  Пж 1 «П»)   Ч ф, ш ч Э Ф   Тп ы ы п   П :       Рб пх (п п ), ч ч пы Оч, п б, ы ы ф (Я), п Вы б ч б Уч б б ч п п цых ( Пж 2 «Б »)       П , б ч, жы б ч, ы ч, пжых п ж Кжы ч б ж, п бх ж пч ц ч ( Пж 3 «З»)       Д х, пш п ш ч, п ч ( Пж 4 «Дпы ч») З п ч п ш Дш х ы ж ч ( Пж 5 «Дш »)   Дш : – Об ыш ч – Ош ч п – У ч п п     Сп ы       Яш, А П п ф х ых пх, 10 – 11 ы: ы, ы, ы, ы п /А Яш – М: Гб; В: П, 2009 – 240 – (Кч бч)     К, ЛА Ф – 19 Ры ы ы бы / ЛА К – М: И, 200 – 19:      О, ВА Ф: З п () / К ЛА – М: И, 200 – 14     М, АЕ Кы бы п ф: 10 – 11 : ч / АЕ М, ЕА М – 3- – М: П, 200 – 11   Т Сч, ч ф шы №3 Кш Т б   В ппых фх – пц, пы ы

Воспитательные задачи:

развивать у обучающихся навыки аккуратности при оформлении задач,  коммуникативности в общении, самостоятельности при практическом применении полученных знаний,  самооценки и самоанализа.

Развивающие: развивать мышление, умение анализировать, сопоставлять, сравнивать и систематизировать, память, внимание.

Цели урока:

– Создание условий для обобщения и закрепления знаний, полученных по теме «Законы Ньютона».

– Совершенствование навыков решения качественных и расчетных задач.

– Расширение  кругозора учащихся, развитие познавательного интереса к предмету.

Тип урока: обобщающий с использованием ИКТ.

Формы деятельности учащихся: фронтальная, работа в парах, индивидуальная.

Оборудование: компьютер, мультимедийный проектор, экран, мобильный класс.

Ход урока.

Учитель: Наш урок начинается с эпиграфа: «Ум заключается не только в знаниях, но и в умении применять знания на деле».

Так давайте попробуем, применив знания, сформулировать тему нашего урока. Для этого разгадаем кроссворд.(см. презентацию, слайды 1-3).

в з а и м о д е й с т в и е                     с и л а                           р а в н о д е й с т в у ю щ а я           м а с с а                       д и Н а м о м е т р                   т р е н и е                                 с к о р о с т ь             к и л о г р а м м

Вопросы:

1.       Действие тел друг на друга.

2.      Векторная физическая величина, которая является причиной возникновения ускорения.

3.      Сила, которая оказывает на тело такое же действие, как несколько сил, взятых вместе.

4.      Скалярная физическая величина, являющаяся мерой инертности тела.

5.      Прибор для измерения силы.

6.      Сила, препятствующая движению одного тела по поверхности другого.

7.      Векторная физическая величина, которая изменяется только в результате воздействия на него силы.

8.      Единицы измерения массы тела.

На основе выделенного слова и букв учащиеся сами формулируют тему урока.

Тема урока: «Динамика. Законы Ньютона»

Главная задача – обобщить знания, полученные при изучении темы «Законы Ньютона»

Учитель: А для этого необходимо вспомнить основные особенности законов Ньютона.

Учащимся необходимо дополнить слайды, предоставленные учителем, отвечая на вопросы.

1.      Дополнить формулировку первого закона Ньютона.

2.      Какое еще название имеет первый закон Ньютона?

3.      Для каких систем справедлив первый закон ?

4.      Дополнить формулировку второго закона Ньютона.

5.      Математическая запись второго закона Ньютона.

6.      Дополнить формулировку третьего закона Ньютона.

7.      С точки зрения третьего закона Ньютона силы возникают?..

(см. презентацию, слайды 4-9)

После работы со слайдами учащиеся получают памятку по теме «Законы Ньютона», которая может помочь им при дальнейшей работе на уроке.

(см.  Приложение 1 «Памятка»)

Человек знает физику, если он умеет решать задачи.

Энрико Ферми

Учитель: Теперь вы готовы применить свои знания на деле.

Последовательность действий:

1.      Работая с тестом в парах (используется программа генератора тестов), учащиеся отвечают на вопросы. Отвечая, заполняют бланк, выставляя свои ответы в графе (Я), а затем проверяют. Выставляют общее количество баллов. Учитель собирает бланки и на доске отмечает степень усвоения материала теста с помощью разноцветных магнитов.

(см. Приложение 2 «Бланки к тесту»)

2.      После того, как работа с тестом закончена, каждый берет с карточку, на которой даны условия задач, расположенных по мере усложнения материала. Каждый ученик работает в своем режиме, при необходимости можно получить консультацию учителя.

(см. Приложение 3 «Задания»)

3.      Для тех, кто успешно справился с решением задач, дается дополнительное задание на отдельной карточке.

(см. Приложение 4 «Дополнительные задачи»)

За пять минут до окончания урока подводятся итоги урока и дается домашнее задание. Домашнее задание находится на отдельном столе и является индивидуальным для каждого ученика.

(см. Приложение 5 «Домашнее задание»)

Домашнее задание составлялось в соответствии с тремя условиями:

– Особенности мышления ученика.

– Отношение ученика к предмету.

– Уровень знаний и умений ученика по предмету.

Список литературы.

1.    Янушевская, Н.А. Повторение и контроль знаний по физике на уроках и внеклассных мероприятиях, 10 – 11 классы: диктанты, тесты, кроссворды, внеклассные мероприятия /Н. А. Янушевская – Москва: Глобус; Волгоград: Панорама, 2009. – 240с – (Качество обучения).

2.    Кирик, Л.А. Физика – 19. Разноуровневые самостоятельные и контрольные работы. / Л.А. Кирик – Москва: Илекса, 2005. – 192с.:ил.

3.     Орлов, В.А. Физика: Задания для самопроверки и контроля с генератором тестов (диск). / Кирик Л.А. – Москва: Илекса, 2008. – 144с.

4.    Марон, А.Е. Контрольные работы по физике: 10 – 11 класс: книга для учителя / А.Е. Марон, Е.А. Марон. – 3- е изд. – Москва: Просвещение, 2005. – 111с.

Татьяна Сочкова, учитель физики средней школы №3 города Кашина Тверской области

В прикрепленных файлах – презентация, дополнительные материалы.

%PDF-1.6 % 1 0 obj > /Metadata 2 0 R /Outlines 3 0 R /Pages 4 0 R /StructTreeRoot 5 0 R /Type /Catalog >> endobj 6 0 obj /Company /CreationDate (D:20170619113929+08’00’) /Creator /Keywords () /ModDate (D:20170703120752+07’00’) /Producer (Adobe PDF Library 10. 0) /Subject () /Title () >> endobj 2 0 obj > stream 2017-07-03T12:07:52+07:002017-06-19T11:39:29+08:002017-07-03T12:07:52+07:00Acrobat PDFMaker 10.1 для Wordapplication/pdf

  • Мерзляков
  • uuid:d25baec3-0668-45e3-96e8-b9f72c3f3580uuid:f333dfe5-48f6-47ef-ab20-0bab7ff91102Adobe PDF Library 10.0 endstream endobj 3 0 obj > endobj 4 0 obj > endobj 5 0 obj > endobj 7 0 obj > endobj 8 0 obj >> endobj 9 0 obj > endobj 10 0 obj > endobj 11 0 obj > endobj 12 0 obj > endobj 13 0 obj > endobj 14 0 obj > endobj 15 0 obj > endobj 16 0 obj > endobj 17 0 obj >> endobj 18 0 obj > endobj 19 0 obj > endobj 20 0 obj > endobj 21 0 obj /I 520 0 R /P 0 >> /Resources > /Font > /ProcSet [/PDF /Text] /XObject > >> /Rotate 0 /Type /Page /Annots [529 0 R] >> endobj 22 0 obj /I 520 0 R /P 1 >> /Resources > /Font > /ProcSet [/PDF /Text] >> /Rotate 0 /Type /Page >> endobj 23 0 obj > /Font > >> /Rotate 0 /StructParents 2 /Type /Page >> endobj 24 0 obj > /Font > >> /Rotate 0 /StructParents 3 /Type /Page >> endobj 25 0 obj > /ExtGState > /Font > /XObject > >> /Rotate 0 /StructParents 4 /Type /Page >> endobj 26 0 obj > /ExtGState > /Font > /XObject > >> /Rotate 0 /StructParents 5 /Type /Page >> endobj 27 0 obj > /Font > >> /Rotate 0 /StructParents 6 /Type /Page >> endobj 28 0 obj > /ExtGState > /Font > /XObject > >> /Rotate 0 /StructParents 7 /Type /Page >> endobj 29 0 obj > /Font > >> /Rotate 0 /StructParents 8 /Type /Page >> endobj 30 0 obj > /ExtGState > /Font > /XObject > >> /Rotate 0 /StructParents 9 /Type /Page >> endobj 31 0 obj > /ExtGState > /Font > /XObject > >> /Rotate 0 /StructParents 10 /Type /Page >> endobj 32 0 obj > /ExtGState > /Font > /XObject > >> /Rotate 0 /StructParents 11 /Type /Page >> endobj 33 0 obj > /Font > >> /Rotate 0 /StructParents 12 /Type /Page >> endobj 34 0 obj > /ExtGState > /Font > /XObject > >> /Rotate 0 /StructParents 13 /Type /Page >> endobj 35 0 obj > /Font > >> /Rotate 0 /StructParents 14 /Type /Page >> endobj 36 0 obj > /ExtGState > /Font > /XObject > >> /Rotate 0 /StructParents 15 /Type /Page >> endobj 37 0 obj > /ExtGState > /Font > /XObject > >> /Rotate 0 /StructParents 16 /Type /Page >> endobj 38 0 obj > /ExtGState > /Font > /XObject > >> /Rotate 0 /StructParents 17 /Type /Page >> endobj 39 0 obj > /ExtGState > /Font > /XObject > >> /Rotate 0 /StructParents 18 /Type /Page >> endobj 40 0 obj > /ExtGState > /Font > /XObject > >> /Rotate 0 /StructParents 19 /Type /Page >> endobj 41 0 obj > /Font > >> /Rotate 0 /StructParents 20 /Type /Page >> endobj 42 0 obj > /Font > >> /Rotate 0 /StructParents 21 /Type /Page >> endobj 43 0 obj > /ExtGState > /Font > /XObject > >> /Rotate 0 /StructParents 22 /Type /Page >> endobj 44 0 obj /I 597 0 R /P 2 >> /Resources > /Font > /ProcSet [/PDF /Text] >> /Rotate 0 /Type /Page >> endobj 45 0 obj > endobj 46 0 obj > endobj 47 0 obj > endobj 48 0 obj > endobj 49 0 obj > endobj 50 0 obj > endobj 51 0 obj > endobj 52 0 obj > endobj 53 0 obj > endobj 54 0 obj > endobj 55 0 obj > endobj 56 0 obj > endobj 57 0 obj > endobj 58 0 obj > endobj 59 0 obj > endobj 60 0 obj > endobj 61 0 obj > endobj 62 0 obj > endobj 63 0 obj > endobj 64 0 obj > endobj 65 0 obj > endobj 66 0 obj > endobj 67 0 obj > endobj 68 0 obj > endobj 69 0 obj > endobj 70 0 obj > endobj 71 0 obj > endobj 72 0 obj > endobj 73 0 obj > endobj 74 0 obj > endobj 75 0 obj > endobj 76 0 obj > endobj 77 0 obj > endobj 78 0 obj > endobj 79 0 obj > endobj 80 0 obj > endobj 81 0 obj > endobj 82 0 obj > endobj 83 0 obj > endobj 84 0 obj > endobj 85 0 obj > endobj 86 0 obj > endobj 87 0 obj > endobj 88 0 obj > endobj 89 0 obj > endobj 90 0 obj > endobj 91 0 obj > endobj 92 0 obj > endobj 93 0 obj > endobj 94 0 obj > endobj 95 0 obj > endobj 96 0 obj > endobj 97 0 obj > endobj 98 0 obj > endobj 99 0 obj > endobj 100 0 obj > endobj 101 0 obj > endobj 102 0 obj > endobj 103 0 obj > endobj 104 0 obj > endobj 105 0 obj > endobj 106 0 obj > endobj 107 0 obj > endobj 108 0 obj > endobj 109 0 obj > endobj 110 0 obj > endobj 111 0 obj > endobj 112 0 obj > endobj 113 0 obj > endobj 114 0 obj > endobj 115 0 obj > endobj 116 0 obj > endobj 117 0 obj > endobj 118 0 obj > endobj 119 0 obj > endobj 120 0 obj > endobj 121 0 obj > endobj 122 0 obj > endobj 123 0 obj > endobj 124 0 obj > endobj 125 0 obj > endobj 126 0 obj > endobj 127 0 obj > endobj 128 0 obj > endobj 129 0 obj > endobj 130 0 obj > endobj 131 0 obj > endobj 132 0 obj > endobj 133 0 obj > endobj 134 0 obj > endobj 135 0 obj > endobj 136 0 obj > endobj 137 0 obj > endobj 138 0 obj > endobj 139 0 obj > endobj 140 0 obj > endobj 141 0 obj > endobj 142 0 obj > endobj 143 0 obj > endobj 144 0 obj > endobj 145 0 obj > endobj 146 0 obj > endobj 147 0 obj > endobj 148 0 obj > endobj 149 0 obj > endobj 150 0 obj > endobj 151 0 obj > endobj 152 0 obj > endobj 153 0 obj > endobj 154 0 obj > endobj 155 0 obj > endobj 156 0 obj > endobj 157 0 obj > endobj 158 0 obj > endobj 159 0 obj > endobj 160 0 obj > endobj 161 0 obj > endobj 162 0 obj > endobj 163 0 obj > endobj 164 0 obj > endobj 165 0 obj > endobj 166 0 obj > endobj 167 0 obj > endobj 168 0 obj > endobj 169 0 obj > endobj 170 0 obj > endobj 171 0 obj > endobj 172 0 obj > endobj 173 0 obj > endobj 174 0 obj > endobj 175 0 obj > endobj 176 0 obj > endobj 177 0 obj > endobj 178 0 obj > endobj 179 0 obj > endobj 180 0 obj > endobj 181 0 obj > endobj 182 0 obj > endobj 183 0 obj > endobj 184 0 obj > endobj 185 0 obj > endobj 186 0 obj > endobj 187 0 obj > endobj 188 0 obj > endobj 189 0 obj > endobj 190 0 obj > endobj 191 0 obj > endobj 192 0 obj > endobj 193 0 obj > endobj 194 0 obj > endobj 195 0 obj > endobj 196 0 obj > endobj 197 0 obj > endobj 198 0 obj > endobj 199 0 obj > endobj 200 0 obj > endobj 201 0 obj > endobj 202 0 obj > endobj 203 0 obj > endobj 204 0 obj > endobj 205 0 obj > endobj 206 0 obj > endobj 207 0 obj > endobj 208 0 obj > endobj 209 0 obj > endobj 210 0 obj > endobj 211 0 obj > endobj 212 0 obj > endobj 213 0 obj > endobj 214 0 obj > endobj 215 0 obj > endobj 216 0 obj > endobj 217 0 obj > endobj 218 0 obj > endobj 219 0 obj > endobj 220 0 obj > endobj 221 0 obj > endobj 222 0 obj > endobj 223 0 obj > endobj 224 0 obj > endobj 225 0 obj > endobj 226 0 obj > endobj 227 0 obj > endobj 228 0 obj > endobj 229 0 obj > endobj 230 0 obj > endobj 231 0 obj > endobj 232 0 obj > endobj 233 0 obj > endobj 234 0 obj > endobj 235 0 obj > endobj 236 0 obj > endobj 237 0 obj > endobj 238 0 obj > endobj 239 0 obj > endobj 240 0 obj > endobj 241 0 obj > endobj 242 0 obj > endobj 243 0 obj > endobj 244 0 obj > endobj 245 0 obj > endobj 246 0 obj > endobj 247 0 obj > endobj 248 0 obj > endobj 249 0 obj > endobj 250 0 obj > endobj 251 0 obj > endobj 252 0 obj > endobj 253 0 obj > endobj 254 0 obj > endobj 255 0 obj > endobj 256 0 obj > endobj 257 0 obj > endobj 258 0 obj > endobj 259 0 obj > endobj 260 0 obj > endobj 261 0 obj > endobj 262 0 obj > endobj 263 0 obj > endobj 264 0 obj > endobj 265 0 obj > endobj 266 0 obj > endobj 267 0 obj > endobj 268 0 obj > endobj 269 0 obj > endobj 270 0 obj > endobj 271 0 obj > endobj 272 0 obj > endobj 273 0 obj > endobj 274 0 obj > endobj 275 0 obj > endobj 276 0 obj > endobj 277 0 obj > endobj 278 0 obj > endobj 279 0 obj > endobj 280 0 obj > endobj 281 0 obj > endobj 282 0 obj > endobj 283 0 obj > endobj 284 0 obj > endobj 285 0 obj > endobj 286 0 obj > endobj 287 0 obj > endobj 288 0 obj > endobj 289 0 obj > endobj 290 0 obj > endobj 291 0 obj > endobj 292 0 obj > endobj 293 0 obj > endobj 294 0 obj > endobj 295 0 obj > endobj 296 0 obj > endobj 297 0 obj > endobj 298 0 obj > endobj 299 0 obj > endobj 300 0 obj > endobj 301 0 obj > endobj 302 0 obj > endobj 303 0 obj > endobj 304 0 obj > endobj 305 0 obj > endobj 306 0 obj > endobj 307 0 obj > endobj 308 0 obj > endobj 309 0 obj > endobj 310 0 obj > endobj 311 0 obj > endobj 312 0 obj > endobj 313 0 obj > endobj 314 0 obj > endobj 315 0 obj > endobj 316 0 obj > endobj 317 0 obj > endobj 318 0 obj > endobj 319 0 obj > endobj 320 0 obj > endobj 321 0 obj > endobj 322 0 obj > endobj 323 0 obj > endobj 324 0 obj > endobj 325 0 obj > endobj 326 0 obj > endobj 327 0 obj > endobj 328 0 obj > endobj 329 0 obj > endobj 330 0 obj > endobj 331 0 obj > endobj 332 0 obj > endobj 333 0 obj > endobj 334 0 obj > endobj 335 0 obj > endobj 336 0 obj > endobj 337 0 obj > endobj 338 0 obj > endobj 339 0 obj > endobj 340 0 obj > endobj 341 0 obj > endobj 342 0 obj > endobj 343 0 obj > endobj 344 0 obj > endobj 345 0 obj > endobj 346 0 obj > endobj 347 0 obj > endobj 348 0 obj > endobj 349 0 obj > endobj 350 0 obj > endobj 351 0 obj > endobj 352 0 obj > endobj 353 0 obj > endobj 354 0 obj > endobj 355 0 obj > endobj 356 0 obj > endobj 357 0 obj > endobj 358 0 obj > endobj 359 0 obj > endobj 360 0 obj > endobj 361 0 obj > endobj 362 0 obj > endobj 363 0 obj > endobj 364 0 obj > endobj 365 0 obj > endobj 366 0 obj > endobj 367 0 obj > endobj 368 0 obj > endobj 369 0 obj > endobj 370 0 obj > endobj 371 0 obj > endobj 372 0 obj > endobj 373 0 obj > endobj 374 0 obj > endobj 375 0 obj > endobj 376 0 obj > endobj 377 0 obj > endobj 378 0 obj > endobj 379 0 obj > endobj 380 0 obj > endobj 381 0 obj > endobj 382 0 obj > endobj 383 0 obj > endobj 384 0 obj > endobj 385 0 obj > endobj 386 0 obj > endobj 387 0 obj > endobj 388 0 obj > endobj 389 0 obj > endobj 390 0 obj > endobj 391 0 obj > endobj 392 0 obj > endobj 393 0 obj > endobj 394 0 obj > endobj 395 0 obj > endobj 396 0 obj > endobj 397 0 obj > endobj 398 0 obj > endobj 399 0 obj > endobj 400 0 obj > endobj 401 0 obj > endobj 402 0 obj > endobj 403 0 obj > endobj 404 0 obj > endobj 405 0 obj > endobj 406 0 obj > endobj 407 0 obj > endobj 408 0 obj > endobj 409 0 obj > endobj 410 0 obj > endobj 411 0 obj > endobj 412 0 obj > endobj 413 0 obj > endobj 414 0 obj > endobj 415 0 obj > endobj 416 0 obj > endobj 417 0 obj > endobj 418 0 obj > endobj 419 0 obj > endobj 420 0 obj > endobj 421 0 obj > endobj 422 0 obj > endobj 423 0 obj > endobj 424 0 obj > endobj 425 0 obj > endobj 426 0 obj > endobj 427 0 obj > endobj 428 0 obj > endobj 429 0 obj > endobj 430 0 obj > endobj 431 0 obj > endobj 432 0 obj > endobj 433 0 obj > endobj 434 0 obj > endobj 435 0 obj > endobj 436 0 obj > endobj 437 0 obj > endobj 438 0 obj > endobj 439 0 obj > endobj 440 0 obj > endobj 441 0 obj > endobj 442 0 obj > endobj 443 0 obj > endobj 444 0 obj > endobj 445 0 obj > endobj 446 0 obj > endobj 447 0 obj > endobj 448 0 obj > endobj 449 0 obj > endobj 450 0 obj > endobj 451 0 obj > endobj 452 0 obj > endobj 453 0 obj > endobj 454 0 obj > endobj 455 0 obj > endobj 456 0 obj > endobj 457 0 obj > endobj 458 0 obj > endobj 459 0 obj > endobj 460 0 obj > endobj 461 0 obj > endobj 462 0 obj > endobj 463 0 obj > endobj 464 0 obj > endobj 465 0 obj > endobj 466 0 obj > endobj 467 0 obj > endobj 468 0 obj > endobj 469 0 obj > endobj 470 0 obj > endobj 471 0 obj > endobj 472 0 obj > endobj 473 0 obj > endobj 474 0 obj > endobj 475 0 obj > endobj 476 0 obj > endobj 477 0 obj > endobj 478 0 obj > endobj 479 0 obj > endobj 480 0 obj > endobj 481 0 obj > endobj 482 0 obj > endobj 483 0 obj > endobj 484 0 obj > endobj 485 0 obj > endobj 486 0 obj > endobj 487 0 obj > endobj 488 0 obj > endobj 489 0 obj > endobj 490 0 obj > endobj 491 0 obj > endobj 492 0 obj > endobj 493 0 obj > endobj 494 0 obj > endobj 495 0 obj > endobj 496 0 obj > endobj 497 0 obj > endobj 498 0 obj > endobj 499 0 obj > endobj 500 0 obj > endobj 501 0 obj > endobj 502 0 obj > endobj 503 0 obj > endobj 504 0 obj > endobj 505 0 obj > endobj 506 0 obj > endobj 507 0 obj > endobj 508 0 obj > endobj 509 0 obj > endobj 510 0 obj > endobj 511 0 obj > endobj 512 0 obj /Company /CreationDate (D:20170619113749+08’00’) /Creator /Keywords () /ModDate (D:20170619113749+08’00’) /Producer (Adobe PDF Library 10. 0) /SourceModified (D:20170512062758) /Subject () /Title () >> endobj 513 0 obj > stream 2017-06-19T11:37:49+08:002017-06-19T11:37:49+08:002017-06-19T11:37:49+08:00Acrobat PDFMaker 10.1 для Worduuid:e851fba9-88d1-45b0-9992-c063639f5c2euuid:3b6b826e-75ff-4c38-a854-0cb962809ea6
  • 4
  • application/pdf
  • Мерзляков
  • Adobe PDF Library 10.0D:20170512062758Личная endstream endobj 514 0 obj /Comments () /Company /CreationDate (D:20170619113809+08’00’) /Creator /Keywords () /MTWinEqns (1) /ModDate (D:20170619113814+08’00’) /Producer (Adobe PDF Library 10.0) /SourceModified (D:20170322012352) /Subject () /Title () >> endobj 515 0 obj > endobj 516 0 obj > stream 2017-06-19T11:38:14+08:002017-06-19T11:38:09+08:002017-06-19T11:38:14+08:00Acrobat PDFMaker 10.1 для Worduuid:2048ec5a-5fb0-4882-9908-aadb94b8cd83uuid:bcbf71d2-a820-4ac6-a568-2dd11521e26e
  • 30
  • application/pdf
  • Мерзляков
  • Adobe PDF Library 10. 0D:20170322012352Личная1 endstream endobj 517 0 obj > stream HWM$5ϯ-JAJvHH8DZTuAt%;۟O`t0~

    Законы движения Ньютона – с примерами, проблемами, решениями и визуализациями

    Учебники по базовой механике – краткий обзор

    Лекции в Массачусетском технологическом институте Уолтера Левина: законы Ньютона


    Законы движения Ньютона

    Целевая аудитория: старшеклассники, первокурсники и второкурсники, студенты, готовящиеся к International Baccalaureate (IB), AP Physics B, AP Physics C, A Level, Singapore / GCE A-Level;

    Студенты классов 11/12 в Индии, готовящиеся к ISC / CBSE и вступительным экзаменам, таким как IIT-JEE / AIEEE. Всем, кому нужен этот учебник в качестве справочного материала!

    Это также может быть полезно при изучении тем, требуемых Common Core Physics.

    а также решения проблем.)

    Введение в законы движения Ньютона

    Наука возникает благодаря наблюдению за природой и выводам из нее с последующим разработкой и проведением экспериментов для проверки или опровержения теорий.Три закона движения, открытые Ньютоном, постоянно управляют движением каждого объекта в природе, но из-за трения и сопротивления воздуха их немного трудно увидеть.


    Первый закон Ньютона сформулирован как:

    «В отсутствие внешних сил покоящийся объект остается неподвижным, а объект в движении продолжает движение с постоянной скоростью (то есть с постоянной скоростью по прямой)».

    Хотя это не то, что мы наблюдаем каждый день. Мяч, катящийся по полу, в конце концов останавливается, быстрее на песчаном полу, чем на мраморном. Это происходит из-за силы трения между мячом и полом. Противодействующая сила в направлении, противоположном его скорости, замедляет мяч и в конечном итоге приводит его в состояние покоя. Если бы мяч катился по полу без трения (идеальный случай), он никогда не остановился бы при отсутствии внешних сил.


    Второй закон Ньютона гласит:

    Ускорение объекта прямо пропорционально действующей на него чистой силе и обратно пропорционально его массе.

    ∑F = ma

    Этот закон немного легче соблюдать по сравнению с первым законом. Вы можете бросить более легкий выстрел дальше, чем более тяжелый, даже если вложите всю свою энергию (или силу) в оба случая. Это происходит потому, что более легкий получает большее ускорение по сравнению с более тяжелым и может преодолеть большее расстояние, прежде чем упасть. Хотя существует множество других факторов, таких как угол броска, сопротивление воздуха и т. Д., Которые определяют расстояние, пройденное выстрелом перед приземлением, но если предположить, что эти факторы эквивалентны для обоих бросков, это должно дать вам некоторое представление о том, что второй Ньютон закон имеет силу.

    Третий закон Ньютона гласит:

    Если два объекта взаимодействуют, сила F12, прикладываемая объектом 1 к объекту 2, равна по величине и противоположна по направлению силе F21, прикладываемой объектом 2 к объекту 1:

    F12 = -F21

    Этот закон можно понять, рассмотрев следующий пример. Когда вы бьете по футбольному мячу босиком, нога болит меньше, если вы ударяете по ней мягко, и болит больше, если вы ударяете по ней с большей силой. Таким образом, футбольный мяч оказывает на вашу ногу столько же силы, сколько и вы.Теперь, когда вы думаете обо всех телах, которые Земля притягивает своей силой гравитации, может возникнуть путаница в отношении этого правила. А что насчет силы, которую они возвращают на землю? Это очень верно, все, что Земля притягивает к себе, также притягивает Землю к себе с силой, равной величине g (ускорение свободного падения), но масса земли настолько велика по сравнению с величиной приложенной силы. что он фактически остается в покое, в то время как тело ускоряется к нему и падает на поверхность земли.


    Темы, которые в учебном пособии будут представлены и применены в различных интересных задачах:

    Построение диаграмм свободного тела и определение сил, действующих на тело

    Решая любую задачу о законах движения Ньютона, мы используем диаграммы свободного тела. На этих диаграммах мы представляем все внешние силы, действующие на объект, а затем применяем второй закон Ньютона, чтобы найти его ускорение и другие параметры.Если анализируемая система включает более одного объекта, то диаграммы их свободных тел строятся отдельно, а затем решаются.

    Трение: Статическое и динамическое трение С силой трения вы сталкиваетесь постоянно, поэтому ее легче всего понять. Сила трения очень интересна тем, что ее величина изменяется до максимального значения в зависимости от внешней силы, приложенной к объекту. Предположим, вы пытаетесь толкнуть тяжелую коробку, чтобы сдвинуть ее в другое место.Сначала вы начинаете толкать его с небольшой силой и продолжаете увеличивать силу, пока она не начнет скользить. Ниже этого значения силы коробка остается в покое независимо от силы, приложенной к ней вами. Схема свободного тела коробки выглядит как
    Для того, чтобы коробка оставалась в равновесии, сила трения всегда должна быть равна прилагаемой вами силе. Отсюда следует вывод, что сила трения (до тех пор, пока коробка не перестанет двигаться) равна приложенной силе, пока не достигнет максимума, после чего она остается постоянной.
    Таким образом, сила трения противодействует (приближающемуся или действительному) относительному движению между двумя контактирующими поверхностями. Статическое трение fs противодействует приближающемуся относительному движению; кинетическое трение fk противостоит действительному относительному движению. Они не зависят от области контакта и удовлетворяют следующим приблизительным законам:

    fs ≤ (fs) max = μsR

    fk =

    мкР μs (коэффициент статического трения) и μk (коэффициент кинетического трения) – константы, характерные для пары контактирующих поверхностей. Экспериментально установлено, что µk немного меньше мкс. Экспериментально установлено, что сила трения не зависит от площади контакта между телами, что также видно из выражений для силы. Статическое трение – это саморегулирующаяся сила до своего предела μsN (fs ≤ μs N). Вы не должны ставить fs = μsN, не будучи уверенным в том, что максимальное значение статического трения вступает в силу. Сила трения, которая противодействует относительному движению между контактирующими поверхностями, называется кинетическим трением или трением скольжения.

    Важные моменты: Во всех задачах о законах движения Ньютона начертите диаграммы свободного тела для каждого объекта в системе отдельно, а затем решите неизвестное.
    Физика (и, вероятно, самая сложная часть) в этих задачах состоит в том, чтобы распознать ограничения, которые связывают разные части системы, например, два объекта должны двигаться с одинаковым ускорением, иначе объект не может потерять контакт с поверхностью склона, поэтому сумма сил, действующих на объект перпендикулярно поверхности, должна быть равна нулю. Отдых – это математика, и он легко приходит после выполнения нескольких задач. Овладев этой способностью, вы сможете решить любую задачу по этой теме.
    Струна имеет одинаковое натяжение во всех точках. Каждая бесконечно малая часть струны имеет одинаковое натяжение, пытаясь развести ее в противоположных направлениях.


    Связанные визуализации

    Вот некоторые из проблем, решенных в этом руководстве:

    Q: Груз весом 5 кг подвешен на веревке длиной 2 м от потолка.Сила 45 Н в горизонтальном направлении прилагается к средней точке R веревки, как показано. Каков угол между веревкой и вертикалью в состоянии равновесия? (Возьмем g = 10 мс-2). Пренебрегайте массой веревки.

    Q: Масса 3 кг опирается на горизонтальную плоскость. Плоскость постепенно наклоняют до тех пор, пока под углом θ = 20 ° к горизонту масса просто не начнет скользить. Какой коэффициент статического трения между блоком и поверхностью?

    Q: Небольшой блок B помещается на другой блок A массой 7 кг и длиной 15 см. Первоначально блок B находится рядом с правым концом блока A. К блоку A прилагается постоянная горизонтальная сила 10 Н. Все поверхности предполагаются без трения. Найдите время, прошедшее до отделения блока B от блока A.

    Q: Шарик массой 5 ​​кг и блок массой 12 кг прикреплены легким шнуром, который проходит через бесфрикционный шкив незначительной массы, как показано на рисунке. Блок лежит на наклоне без трения под углом 30o. Найдите величину ускорения двух предметов и натяжение шнура.Возьмем g = 10 мс-2.

    Q: Два блока массой 5 ​​кг и 7 кг помещаются в контакт друг с другом на горизонтальной поверхности без трения. К блоку массой 7 кг прилагается постоянная горизонтальная сила 20 Н.
    (a) Определите величину ускорения двухблочной системы.

    (b) Определите величину силы контакта между двумя блоками.

    Q: Блок массой 5 ​​кг начинает скользить по плоскости без трения, имеющей наклон 25. 0 ° от места покоя вверху. Длина уклона составляет 2,00 м. Найдите (а) ускорение блока и (б) его скорость, когда он достигает нижней части уклона. Возьмем g = 10 мс-2.

    Q: Мужчина весом 75,0 кг стоит на платформенных весах лифта. В состоянии покоя лифт поднимается вверх, достигая максимальной скорости 1,20 м / с за 1,00 с. Следующие 10,00 с он движется с этой постоянной скоростью. Затем лифт испытывает равномерное ускорение в отрицательном направлении оси y в течение 1.70 с и приходит в состояние покоя. Что регистрируют весы
    (a) перед началом движения лифта?
    (б) в течение первых 1.00 с?
    (c) при движении лифта с постоянной скоростью?
    (d) во время замедления? Возьмем g = 10 мс-2.

    Q: Блок массой 2,50 кг ускоряется по шероховатой поверхности с помощью веревки, проходящей через шкив, как показано на рисунке. Натяжение троса составляет 12,0 Н, а шкив находится на 10,0 см выше верха блока.Коэффициент кинетического трения 0,300. (а) Определите ускорение блока при x = 0,400 м. (b) Найдите значение x, при котором ускорение становится равным нулю. Возьмем g = 10 мс-2.

    Q: Блоку придается начальная скорость 5,00 м / с на уклоне 20,0 ° без трения. Насколько высоко по склону сдвигается блок, прежде чем остановиться? Возьмем g = 10 мс-2.

    Q: Два блока массой 5 ​​кг и 9 кг соединены струной с незначительной массой, которая проходит по шкиву без трения.Подъемы без трения. Найдите (а) величину ускорения каждого блока и (б) натяжение струны. Возьмем g = 10 мс-2.

    Q: На рисунке блоки A и B имеют массу 40 кг и 20 кг соответственно. (a) Определите минимальный вес блока C для предотвращения скольжения A, если коэффициент трения, мкс между A и столом равен 0,18. Блок C внезапно поднимается с A. (b) Каково ускорение блока A, если μk между A и столом составляет 0,15.Возьмем g = 10 мс-2.

    Q: Две гири 5 кг и 7 кг, расположенные на горизонтальной поверхности без трения, соединены легкой нитью. На одну из масс справа действует сила 50 Н. Определите ускорение системы и натяжение Т в струне.

    Q: Блок массой m1 = 5 кг на горизонтальном столе без трения соединен с блоком массой m2 = 3 кг с помощью очень легкого шкива P1 и легкого неподвижного шкива P2, как показано на рисунке.Если a1 и a2 – ускорения m1 и m2 соответственно, а) какова связь между этими ускорениями? Найдите (b) натяжения в струнах и (c) ускорения a1 и a2. Возьмем g = 10 мс-2.

    Q: Блок A массой 0,5 кг может скользить по наклонной поверхности без трения под углом 30o и длиной 0,8 м, удерживаемой внутри лифта, поднимающегося с равномерной скоростью 2 м / с. Найдите время, за которое блок проскальзывает вниз по длине уклона, если он спускается с вершины уклона.Возьмем g = 10 мс-2.

    Q: Показанный лифт опускается с постоянным ускорением 1,5 м / с. Масса блока А – 1 кг. Какую силу оказывает блок A на блок B? Возьмем g = 10 мс-2.

    Q: Рассмотрим три связанных объекта, показанных на рисунке. Если наклонная плоскость не имеет трения и система находится в равновесии, найдите (в единицах m, g и θ (а) массу M и (b) напряжения T1 и T2. Если значение M в два раза больше найденного значения в части (a) найдите (c) ускорение каждого объекта и (d) напряжения T1 и T2.

    Q: В приведенной выше задаче, если коэффициент трения покоя между м и 2 м и наклонной плоскостью равен мкс, а система находится в равновесии, найдите (а) минимальное значение М и (б) максимальное стоимость М.

    Q: Масса M = 16 кг удерживается на месте приложенной силой F и системой шкивов, как показано на рисунке. Шкивы безмассовые и без трения. Найдите (а) натяжение в каждом участке каната, Т1, Т2, Т3, Т4 и Т5, и (б) величину F.Возьмем g = 10 мс-2.

    Q: Какая горизонтальная сила должна быть приложена к тележке, показанной на рисунке, чтобы блоки оставались неподвижными относительно тележки? Предположим, что все поверхности, колеса и шкив не имеют трения.

    Q: Блок скользит с постоянной скоростью по наклонной плоскости, имеющей угол наклона θ. Затем блок проецируется на ту же плоскость с начальной скоростью u. а) Как далеко он продвинется вверх по самолету, прежде чем остановится? (б) После того, как блок остановится, будет ли он снова скользить по плоскости?

    Q: Блок массы m1 = 4.Сверху на блок массой m2 = 5,0 кг кладут 0 кг. Чтобы верхний блок соскользнул по нижнему, в то время как нижний зафиксирован, к верхнему блоку необходимо приложить горизонтальную силу не менее 12 Н. Теперь сборка блоков размещается на горизонтальном столе без трения. Найдите величины (а) максимальной горизонтальной силы, которая может быть приложена к нижнему блоку, чтобы блоки перемещались вместе, и (б) результирующее ускорение блоков. Возьмем g = 10 мс-2.

    Q: Найдите ускорение блока массы на показанном рисунке.Все поверхности без трения, шкивы и струна легкие.

    Q: Найдите ускорение 5-килограммового блока на рисунке. Возьмем g = 10 мс-2

    Q: На данном рисунке предположим, что m2 = 2,5 кг и m3 = 3 кг. Какой должна быть масса m1, чтобы она оставалась в покое? Возьмем g = 10 мс-2.

    Q: Постоянная сила F = m2g / 3 применяется к массе m1, как показано на рисунке. Шкив и шнурок легкие, а поверхность стола гладкая.Найдите ускорение m1.

    Q: Блок 1 массы m1 = 2,0 кг и блок 2 массы m2 = 1,0 кг соединены нитью с незначительной массой. Блок 2 толкается силой 25 Н и углом θ = 35 °. Коэффициент кинетического трения между каждым блоком и горизонтальной поверхностью составляет 0,25. Какое натяжение струны? Возьмем g = 10 мс-2.

    Q: Два блока, показанные на рисунке, не прикреплены друг к другу. Коэффициент трения покоя между блоками μs = 0.35, но поверхность под большим блоком не имеет трения. Какая минимальная величина горизонтальной силы требуется, чтобы меньший блок не соскользнул с большого блока? Возьмем g = 10 мс-2.

    Q: Найдите массу подвесного блока на рисунке, которая предотвратит проскальзывание меньшего блока по треугольному блоку. Все поверхности без трения, струны и шкив легкие.

    Руководство с решенными задачами и примерами:

    Тест MCQ № 1

    Законы Ньютона: некоторые основные проблемы

    Companion MCQ Quiz – проверьте, насколько хорошо вы знаете эту тему. Ваша оценка будет отправлена ​​вам по электронной почте на указанный вами адрес.

    Тест MCQ: законы Ньютона (основные проблемы)

    Google Spreadsheet Form

    Тест MCQ № 2 Законы Ньютона: еще несколько проблем Проверка основ
    Google Spreadsheet Form

    Google Spreadsheet Form


    Тест MCQ № 3


    Законы Ньютона: некоторые сложные проблемы

    Companion MCQ Quiz – проверьте, насколько хорошо вы знаете эту тему. Ваша оценка будет отправлена ​​вам по электронной почте на указанный вами адрес.

    Законы Ньютона – сложные проблемы

    Google Spreadsheet Form


    Веб-сайт класса физики

    Законы движения Ньютона: набор задач

    Проблема 1:

    Африканский слон может достигать высоты 13 футов и иметь массу до 6000 кг.Определите вес африканского слона в Ньютонах и фунтах. (Дано: 1,00 N = 0,225 фунта)

    Задача 2:

    Около двадцати процентов представителей Национальной футбольной лиги весит более 300 фунтов. При этом весе их индекс массы тела (ИМТ) относит их к ожирению 2 степени, что на одну ступень ниже патологического ожирения. Определите массу 300-фунтового (1330 Н) футболиста.

    Задача 3:

    С ростом цен на топливо для автомобилей с горючими двигателями исследователи и производители стали уделять больше внимания концепции сверхлегкого автомобиля. Используя углеродные композиты, более легкие стали и пластмассы, можно изготовить экономичный автомобиль массой 540 кг. Насколько меньше весит сверхлегкий автомобиль по сравнению с 1450-килограммовым Honda Accord (2007 г.)?

    Задача 4:

    По данным Национального центра статистики здравоохранения, средняя масса взрослого американского мужчины составляет 86 кг. Определите массу и вес 86-килограммового человека на Луне, где гравитационное поле в шесть раз меньше земного.

    Задача 5:

    Растущая обеспокоенность спортивных тренеров и защитников здоровья (и родителей) сотрясениями мозга и множественными сотрясениями мозга среди школьных футболистов побудила к многочисленным исследованиям эффективности защитных головных уборов, а также сил и ускорений, испытываемых игроками.Одно исследование показало, что вероятность сотрясения мозга составляет 50% при ударах с ускорением 75 g (т.е. 75 умноженных на 9,8 м / с / с). (Средний удар головой приводит к ускорению от 22 до 24 g. ) Если масса головы игрока (со шлемом) составляет 6,0 кг и считается свободным телом , то какая чистая сила потребуется для создания ускорения 75 g’s?

    Задача 6:

    Капитан ВВС США Джон Стэпп испытал человеческие пределы ускорения, проехавшись на ракетных салазках собственной конструкции, известных как Gee Whiz.Какая полезная сила потребуется для разгона 82-кг Stapp со скоростью 450 м / с / с (максимальное ускорение, испытанное Stapp)?

    Задача 7:

    София, масса которой составляет 52 кг, испытала на себе действующую силу 1800 Н в нижней части петли американских горок во время школьной экскурсии по физике в местный парк развлечений. Определите ускорение Софии в этом месте.

    Задача 8:

    В круизном корабле Top Thrill Dragster в парке развлечений Cedar Point в Огайо используется гидравлическая система запуска для ускорения гонщиков с 0 до 54 м / с (120 миль / ч) за 3 секунды.За 8 секунд до подъема на полностью вертикальный холм высотой 420 футов. Определите чистую силу, необходимую для ускорения человека весом 86 кг.

    Задача 9:

    а. Определите чистую силу, необходимую для разгона сверхлегкого автомобиля массой 540 кг с 0 до 27 м / с (60 миль в час) за 10,0 секунд.
    г. Определите чистую силу, необходимую для разгона Ford Expedition массой 2160 кг с 0 до 27 м / с (60 миль / ч) за 10,0 секунд.

    Задача 10:

    Anna Litical и Noah Formula экспериментируют с влиянием массы и чистой силы на ускорение лабораторной тележки.Они определили, что чистая сила F заставляет тележку массой M ускоряться со скоростью 48 см / с / с. Каково значение ускорения тележки с…

    а. массой M при действии на него чистой силы 2F ?
    г. масса 2M при действии на него чистой силы F ?
    г. масса 2M при действии на него чистой силы 2F ?
    г. масса 4M при действии на него чистой силы 2F ?
    эл. масса 2M при действии на него чистой силы 4F ?

    Задача 11:

    Итан тащит мешок с травой из гаража на улицу вечером накануне вывоза мусора. Диаграмма справа – это диаграмма свободного тела. Он использует стрелки для обозначения сил, действующих на сумку. Каждая сила обозначена в соответствии с типом. Величина силы представлена ​​размером стрелки.Используйте диаграмму свободного тела, чтобы определить чистую силу, действующую на мешок. Значения отдельных сил:

    F grav = F norm = 60,5 N
    F приложение = 40,2 Н
    F frict = 5,7 Н.
    Задача 12:

    К сожалению для Ванессы, колеса ее чемодана не работают. Она натягивает ремешок, пытаясь вывести его из состояния покоя, и тянет к стойке регистрации у тротуара. На диаграмме свободного тела справа показаны силы, действующие на чемодан.Используйте значения силы, чтобы определить чистую силу, массу и ускорение чемодана. Значения отдельных сил:

    F grav = F norm = 207 N
    F десятки = 182 Н
    F frict = 166 Н.
    Задача 13:

    Прибыла партия новых материалов для физики. Их размещают на грузовом лифте и транспортируют на третий этаж для доставки в кабинеты физики. На диаграмме свободного тела справа показаны силы, действующие на грузовой лифт, когда он начинает подъем по шахте лифта.Используйте значения силы, чтобы определить чистую силу, массу и ускорение лифта. Значения отдельных сил:

    F десятков = 2340 N
    F грав = 2120 Н
    F norm1 = F norm2 = 276 N.
    Задача 14:

    Сегодня вечер пятницы, и Скайлеру поручено присмотреть за Кейси, его двухлетним братом, нянькой. Он надевает защитный шлем на Кейси, помещает его в красный фургон и берет на прогулку по окрестностям.Когда Скайлер начинает через улицу, он воздействует на фургон с силой 52 Н. Существует сила сопротивления 24 Н, а общий вес вагона и Кейси составляет 304 Н. Постройте диаграмму свободного тела, показывающую типы сил, действующих на вагон. Затем определите чистую силу, массу и ускорение вагона.

    Задача 15:

    После начального сингла в 8 -м иннинге Эрл пытается украсть вторую базу. Когда он попадает в грязь на голове первым, его 73.Тело весом 2 кг испытывает силу трения 249 Н. Постройте диаграмму свободного тела, изображающую типы сил, действующих на Эрла. Затем определите чистую силу и ускорение.

    Задача 16:

    Мира и Тарик являются партнерами лаборатории Pulley and Bricks. Они определили, что кирпич весом 2,15 кг испытывает силу прямого натяжения 9,54 Н и силу трения 8,69 Н при ускорении по поверхности стола. Постройте диаграмму свободного тела, изображающую типы сил, действующих на кирпич. Затем определите чистую силу и ускорение кирпича.

    Задача 17:

    Через несколько мгновений после принятия ужасного решения выпрыгнуть из двери самолета, тело Дарина весом 82,5 кг испытывает сопротивление воздуха 118 Н. Определите ускорение Дарина в данный момент времени. ПОДСКАЗКА: начните с рисования диаграммы свободного тела и определите чистую силу.

    Задача 18:

    Келли и Джарвис – члены театральной бригады Варьете. Между выступлениями они должны быстро вывести на сцену детский рояль.После того, как занавес закрывается, они прилагают внезапную силу вперед в 524 Н, чтобы вывести пианино из состояния покоя и набрать скорость. Фортепиано весом 158 кг испытывает трение 418 Н.

    а. Каково ускорение фортепиано во время этой фазы движения?
    г. Если Келли и Джарвис сохранят эту силу движения вперед в течение 1,44 секунды, то какой скоростью будет пианино?

    Задача 19:

    Туннели для прыжков с парашютом стали популярными аттракционами, отчасти привлекательными для тех, кто хотел бы испытать опыт прыжков с парашютом, но слишком переполнен страхом выпрыгнуть из самолета на высоте нескольких тысяч футов. Туннели для прыжков с парашютом – это вертикальные аэродинамические трубы, через которые воздух обдувается с высокой скоростью, что позволяет посетителям испытать полеты на теле. Во время первого приключения Натальи внутри туннеля она меняет ориентацию, и на мгновение ее 46,8-килограммовое тело испытывает восходящую силу сопротивления воздуха в 521 Н. Определите ускорение Натальи в этот момент времени.

    Задача 20:

    С помощью веревки из глубокого колодца вытаскивают ведро с водой весом 2,89 кг.

    а. Каково ускорение ковша при натяжении троса 30,2 Н?
    г. Если начать из состояния покоя, какая скорость будет у ковша после воздействия этой силы в течение 2,16 секунды?

    Задача 21:

    Ракета массой 0,104 кг на взлете ускоряется со скоростью 45,9 м / с / с. Определите восходящую тягу, которую испытывает ракета.

    Задача 22:

    Брэндон – ловец бейсбольной команды Университета. Он воздействует на 0. 145-килограммовый бейсбольный мяч, чтобы остановить его со скорости 38,2 м / с. При этом его рука отскакивает на расстояние 0,135 м. Определите ускорение мяча и силу, приложенную к нему Брэндоном.

    Задача 23:

    Алехандра пытается тащить своего 32,6-килограммового золотистого ретривера по деревянному полу, применяя горизонтальную силу. Какую силу она должна приложить, чтобы переместить собаку с постоянной скоростью 0,95 м / с? Коэффициент трения между собакой и полом равен 0.72.

    Задача 24:

    Коэффициент трения между колесами Ford Coupe Доусона 1985 года и сухим асфальтом составляет 0,85. Определите ускорение, которое испытывает купе массой 1300 кг при заносе до полной остановки.

    Задача 25:

    Николас, Брианна, Дилан и Хлоя тренируются в хоккее на замерзшем озере Блюберд. Когда Дилан и Хлоя преследуют шайбу весом 0,162 кг, она замедляется с 10,5 м / с до 8,8 м / с за 14 секунд.

    а. Определите ускорение шайбы.
    г. Определите силу трения, испытываемую шайбой.
    г. Определите коэффициент трения между льдом и шайбой.

    Задача 26:

    Без ведома большинства учеников, каждый раз, когда полы в школе натирают воском, учителя физики собираются вместе, чтобы провести эксперименты по трению в носках с бочкой фуна (ммм – на них есть одежда; просто у них нет обуви на ногах).В одном случае г-н Лондон применяет горизонтальную силу, чтобы ускорить г-на Шнайдера (масса 84 кг) вправо со скоростью 1,2 м / с / с. Если коэффициент трения между носками Шнайдера и свежевыполированным полом составляет 0,35, то с какой силой (в Ньютонах) мистер Лондон должен тянуть?

    Задача 27:

    Декстер Эйус бежит через кафетерий, поскользнулся на картофельном пюре и упал на пол. (Пусть это будет уроком для Декстера.) Декстер приземляется в луже молока и останавливается с ускорением -4.8 м / с / с. Декстер весит 780 Ньютонов. Определите коэффициент трения между декстером и полом молочного цвета .

    Задача 28:

    Harrier Jump Jet – это военный реактивный самолет с неподвижным крылом, предназначенный для вертикального взлета и посадки (VTOL). Он способен поворачивать свои реактивные двигатели из горизонтального положения в вертикальное для взлета, посадки и выполнения горизонтальных маневров. Определите вертикальную тягу, необходимую для разгона 8600-кг Harrier вверх со скоростью 0,40 м / с / с.

    Задача 29:

    Во время прыжков с парашютом Ди Селерат открывает свой парашют и свой 53-й парашют.Тело весом 4 кг мгновенно ускоряется вверх со скоростью 8,66 м / с / с. Определите восходящую силу, которую Ди испытывает в этот момент.

    Задача 30:

    Автомобиль массой 1370 кг трогается с места по горизонтальной поверхности. Автомобиль замедляется с 27,6 м / с до состояния покоя за 3,15 секунды. Предполагая незначительное сопротивление воздуха, определите коэффициент трения между шинами автомобиля и дорожным покрытием.

    Вернуться к обзору

    См.

    Аудиогид решения проблемы: 1 | 2 | 3 | 4 | 5 | 6 | 7 | 8 | 9 | 10 | 11 | 12 | 13 | 14 | 15 | 16 | 17 | 18 | 19 | 20 | 21 | 22 | 23 | 24 | 25 | 26 | 27 | 28 | 29 | 30

    Как решать проблемы с помощью второго закона Ньютона

    Начало

    Расписание

    Занятия

    Материалы

    Интерактивная физика

    Профессиональные

    Личные / Музыкальные

    Ссылки

    Предыдущая страница

    Вот систематический, 10-шаговый метод, который поможет вам пройти 90% виртуально любая проблема, связанная с использованием Второго закона Ньютона.Решение в следующем примере задачи будут показаны шаги метода :

    А 30 чемодан кг поднимается вверх под углом 20 градусов под воздействием приложенной горизонтальной силы 100 Н. Кинетический коэффициент трение между наклоном и чемоданом составляет 0,20. Найдите ускорение чемодана.

    Шаг 1 : Постройте большую схематическую диаграмму физической ситуации.Во время чтения и перечитав формулировку задачи, создайте свою диаграмму, включая все часть информации, которую вы можете извлечь из утверждения на диаграмме. Прикрепите соответствующие символы к каждому важному параметру в задаче, известно значение параметра или нет. Сделайте прямые линии прямыми, параллельные линии параллельны, перпендикулярные линии перпендикулярны и т. д. в меру своих возможностей, чтобы в дальнейшем избежать путаницы.
    Пример справа. (Обратите внимание, как следующие элементы, извлеченные из отчета проблемы были переведены в конкретные элементы на чертеже: «… чемодан весом 30 кг … движется вверх … наклоняется на 20 градусов … приложенная горизонтальная сила 100 Н … кинетический коэффициент трения … 0,20. “)

    Шаг 2: Выберите «систему», к которой вы собираетесь применить Секунду Ньютона. Закон.В некоторых задачах может быть более одного кандидата на «систему». Вы не можете выбрать лучший с первого раза. Без проблем; просто выберите другой и сделайте это снова.
    Пример: Мы выберем чемодан в качестве нашей системы, потому что это то, что нужно к которому прилагаются многие очевидные силы, и это то, что ускорение, которое мы хотим найти.

    Шаг 3: Определите все силы, действующие на «систему».”Сделай это нарисовать пунктирную линию вокруг системы, выбранной на шаге 2, и обозначить все физические объекты , которые контактируют с системой. Каждый из они будут оказывать давление на систему. Затем ищите “поле” силы – силы, которые действуют , не касаясь через посредника поля какого-то. Во вводной механике единственное «поле» сила – это сила тяжести. Это сила, проявляемая землей (или другое очень массивное тело) на систему через гравитационное поле.
    ВАЖНО! Каждые усилие на система прилагает к какой-то физического объекта вне системы. Если вы не можете определить это объект и способ взаимодействия (контакт или поле), сила ДЕЙСТВУЕТ НЕ СУЩЕСТВУЕТ!

    Следующие Вот некоторые часто встречающиеся силы и несколько советов по борьбе с ними:

    • Канаты или струны Они оказывают на систему силы «натяжения».Они всегда направлены на от системы по направлению удлинения веревки или веревки.
    • Контакты с поверхностями Обычно мы разрушаем силу из-за контакта с поверхностью на два компонента, называемых “нормальными”, что означает «перпендикулярный» – сила и «трение» сила. Нормальная сила – это, как правило, сила толчка. направлен в сторону системы, если поверхность не липкая, что позволяет это для приложения силы типа «притяжения». Сила трения равна параллельно поверхности, противостоит движению или потенциальному движению (т.е. система на грани “скольжения”) и является часто считается, что он связан с нормальной силой через коэффициент трения “. Подробнее см. в тексте.
    • Петли или Штифты Они создают силы произвольной величины и направление, необходимое для обеспечения того, чтобы точка крепления не двигаться .
    • Общие толкает или тянет Если проблема указывает , что какой-то объект толкается или тянется в каком-то направлении, вы можете предположить, что указанная сила прилагается каким-либо физическим объектом. Не забывай включить его.
    • Воздух сопротивление Воздух может не быть видно , но очень вероятно физически связывается с вашей системой. Часто мы пренебрегаем сопротивление воздуха, поскольку его влияние считается незначительным. Однако если проблема указывает на определенное сопротивление воздуха или сообщает вам что сопротивление воздуха так или иначе зависит от скорости или других параметров, не забудьте включить его.
    • Гравитационный force Сила тяжести, обычно называемая «весом» системы – это единственная сила, которая действует на без физически связь с системой (по крайней мере, пока вы не узнаете об электрических и магнитных силы позже.) Он действует в нисходящем направлении (по определению !) и равна массе системы, умноженной на локальную гравитационную напряженность поля г – обычно, но ошибочно называют ускорение силы тяжести.”
    Пример справа. Мы находим два объекта, контактирующих с системой – один поверхность и один «толкатель». Таким образом, мы находим в общей сложности четыре силы – нормальная сила и сила трения (от поверхности), толчок (от толкатель), и вес (из-за единственной силы – пока – что действует без необходимости прикосновения – гравитация.)

    Шаг 4: Нарисуйте «диаграмму свободного тела». Система может быть представлена простой круг или квадрат; хотим акцентировать внимание на силах на и результирующее ускорение из системы. Нарисуйте каждую силу с помощью его хвост на поверхности системы простирается в правильном направлении. Включите также вектор ускорения, но отличите его от силы векторов, нарисовав их в другой форме.
    Пример справа. Обратите внимание, что нормальная сила направлена ​​перпендикулярно к поверхность (не показана на схеме свободного тела) сила трения направлена ​​против направления скольжения системы относительно к поверхности, толчок идет в заданном направлении, а вес равен направлен «вниз». Мы показываем ускорение в другом виде вектор, направленный вверх по плоскости, но мы не знаем , это наверняка; он может быть направлен вниз по уклону.Мы напомните себе об этом факте, поставив “(?)” рядом с вектор ускорения.

    Шаг 5: Выберите систему координат и определите углы, по которым силы и ускорения сделать с осями координат. Обычно «умно» подбирать система координат, которая минимизирует количество неизвестных векторов, которые будут должны быть разбиты на компоненты. Ответы, которые вы получите , должны быть и будет независимо от вашего выбора системы координат, но разумный выбор приведет к упрощению решения уравнений.Вы можете нужно сделать геометрическую царапину на другом листе бумаги, чтобы изобразить выяснить, как углы соотносятся с углами, указанными в постановке задачи.
    Пример справа. Мы выбрали систему координат, которая требует от нас определить составляющие только веса и толчка – две силы о которых мы много знаем. Оба они лежат под углом тета (учитывая в постановке задачи как 20 градусов) от одного из направлений оси.

    Шаг 6: Напишите второй закон Ньютона. Это основной физический принцип, который вы вызываете; «отправная точка» для ваших расчетов. Просто сделать это!
    Пример:

    Шаг 7: Примените основное уравнение к этой проблеме. Просто напишите, какая “сумма сил “в данном случае. Если ускорение равно нулю, используйте этот факт чтобы также упростить уравнение.
    Пример:

    Шаг 8: Напишите уравнения компонентов. Это , просто – это вопрос распознавания. что каждое векторное уравнение является сокращением для двух (или, в более общем смысле, три) скалярные уравнения. Просто перепишите векторное уравнение для каждого компонента направление с каждой векторной величиной, переписанной как соответствующий компонент.
    Пример:

    Шаг 9: Определите, что представляет собой каждый компонент с точки зрения амплитуды вектора и тригонометрии. функции присоединенных углов.На этом этапе мы явно указываем знаки компонент вектора. Это также хорошее время, чтобы явно замените “мг” на “W” , если вам известно масса системы
    Пример: Обратите внимание, что нормальная сила составляет чисто в направлении + y, сила трения составляет чисто в направлении -x, толчок имеет положительный x-компонент и отрицательный y-компонент, вес имеет отрицательный x- и y-компоненты, и предполагаемое ускорение составляет чисто дюймов направление + x.Таким образом, мы имеем:

    Шаг 10: Упростите полученные уравнения и выясните, куда двигаться дальше. Этот это конец «метода».
    Пример:

    Теперь вы самостоятельно. Метод сделал для вас поставку набора отношений между величинами различных сил, которые требуются чтобы удовлетворить второму закону Ньютона.Теперь у вас есть “информация” у вас не было раньше в форме этих уравнений. В некоторых проблемах вы можете получить дополнительных информации, применив метод к другая “система”. Что вы делаете с информацией будет зависеть от специфики проблемы. Ниже мы заканчиваем образец проблема.
    Дополнительно к компонентным уравнениям имеем следующую связь между сила трения и нормальная сила:

    Итак, решение уравнение первого компонента для a, используя указанное выше соотношение для f, и подставив из второго компонентного уравнения для n, получим

    С a – , предположительно, – это величина ускорения. и поскольку величины векторов равны , всегда неотрицательны, это ответ говорит нам, что мы выбрали неправильное направление для a; это должно на самом деле быть направленным вниз по склону, что означает, что чемодан замедляется вниз .Ответ

    Законы движения Ньютона – проблемы и решения

    Первый закон движения Ньютона

    1. Если на объект не действует чистая сила, то:

    (1) объект не разгоняется

    (2) объект в состоянии покоя

    (3) изменение скорости объекта = 0

    (4) объект не может двигаться с постоянной скоростью

    Какое утверждение верно.

    Решение

    Правильная выписка:

    (1) Объект не разгоняется

    Чистая сила вызывает ускорение объекта. Таким образом, если нет чистой силы, то объекты не ускоряются.

    (2) Объект в покое

    Первый закон движения Ньютона гласит, что если на объект не действует никакая результирующая сила, то объект всегда находится в состоянии покоя или объект всегда движется с постоянной скоростью.

    (3) Изменение скорости объекта = 0

    Изменение скорости = ускорение.Отсутствие изменения скорости означает отсутствие ускорения. Если ускорения нет, то на объект не действует никакая результирующая сила.

    Объект в лифте

    2. Вес человека в лифте в состоянии покоя = 500 Н. Ускорение свободного падения 10 м / с 2 . Когда лифт ускоряется, сила натяжения составляет 750 Н. Что такое ускорение лифта.

    Известный:

    Вес человека (w) = 500 Ньютон = 500 кг · м · с –2 (подъем в состоянии покоя)

    Ускорение свободного падения (g) = 10 м с –2

    Масса человека (м) = 500/10 = 50 кг

    Сила натяжения (Т) = 750 Н (подъем с ускорением)

    Масса лифта игнорируется.

    Разыскивается: Разгон лифта

    Решение:

    Лифт в состоянии покоя, без разгона (a = 0). Сила, действующая вверх, имеет знак плюса, а сила, действующая вниз, – знак минус.

    ΣF = m a

    Т – ш = 0

    T = w

    T = 500 Ньютон

    Если лифт ускоряется вниз, сила натяжения меньше 500 Н. В противном случае, если лифт ускоряется вверх, сила натяжения превышает 500 Н.

    Сила натяжения = 750 Н, потому что лифт ускоряется вверх. Сила, действующая вверх, имеет знак плюса, а сила, действующая вниз, – знак минус.

    T – ш = м а

    750 – 500 = 50 а

    250 = 50 а

    а = 250/50

    a = 5,0 м с –2

    3. Человек весом 60 кг в лифте ускорился вниз со скоростью 3 м / с 2 . Если ускорение свободного падения составляет 10 м / с 2 , какова нормальная сила, прикладываемая полом лифта к человеку.

    Известный:

    Масса (м) = 60 кг

    Ускорение человека и лифта (а) = 3 м / с 2

    Ускорение свободного падения (g) = 10 м / с 2

    Вес (w) = m g = (60) (10) = 600 Ньютон

    Требуется: Нормальная сила (Н)

    Решение:

    На человека в лифте действуют две силы: вес (w) человека и нормальная сила (N), прилагаемая полом к ​​человеку.Существуют три векторные величины: вес (w), нормальная сила (N) и ускорение лифта, при этом вес действует вниз, нормальная сила действует вверх, а ускорение лифта – вниз. Векторные величины, действующие вниз, имеют знак плюса, а векторные величины, действующие вверх, имеют знак минус.

    ∑F = м

    w – N = (60) (3)

    600 – N = 180

    N = 600 – 180

    N = 420 Ньютон

    4. 40-килограммовый предмет в лифте ускорился вверх.Если пол лифта воздействует на объект 520 Н и ускорение свободного падения составляет 10 м / с 2 . Какое ускорение у лифта?

    Известный:

    Масса (м) = 40 кг

    Нормальная сила (Н) = 520 Н

    Ускорение свободного падения (g) = 10 м / с 2

    вес (w) = m g = (40) (10) = 400 N

    Разыскивается: Разгон лифта

    Решение:

    ∑F = м

    400 – 520 = (40) (а)

    -120 = (40) (а)

    а = -120/40

    a = -3 м / с 2

    Ускорение лифта 3 м / с 2 .Знак минус означает, что лифт движется вверх.

    5. Объект весом 60 кг в лифте ускорялся вниз со скоростью 3 м / с 2 . Какую силу оказывает объект на полу лифта.

    Известный:

    Масса (м) = 60 кг

    Вес (w) = m g = (60 кг) (10 м / с 2 ) = 600 кг м / с 2 = 600 Ньютон

    Ускорение лифта (а) = 3 м / с 2 , вниз

    Разыскивается: Сила, приложенная предметом на полу лифта.

    Решение:

    Лифт ускорялся вниз со скоростью 3 м / с . Сила, действующая вниз, имеет знак плюса, а сила, действующая вверх, – знак минус.

    w – N = m a

    N = ш – м а

    N = 600 – (60) (3)

    N = 600 – 180

    N = 420 Ньютон

    Сила, прилагаемая предметом к этажу лифта = 420 Н.

    6. Два блока соединены шнуром, проходящим через шкив. Не обращайте внимания на массу шнура и шкива и любое трение в шкиве. Масса блока А составляет 6 кг, а масса блока Б – 2 кг. Ускорение свободного падения 10 м / с 2 . Что такое сила натяжения?

    Известный:

    м A = 6 кг, м B = 2 кг, g = 10 м / с 2

    w A = m A g = (6 кг) (10 м / с 2 ) = 60 кг м / с 2

    w B = m B g = (2 кг) (10 м / с 2 ) = 20 кг м / с 2

    Требуется: усилие натяжения (Т)?

    Решение:

    w A > w B , так что m A движется вниз, m B движется вверх.

    Второй закон Ньютона:

    ΣF = m a

    w A – w B = (m A + m B ) a

    60-20 = (6 + 2) а

    40 = (8)

    a = 40/8 = 5 м / с 2

    Сила натяжения:

    м A движется вниз:

    w A – T A = m A a

    60 – Т А = (6) (5)

    60 – Т А = 30

    T A = 60-30

    T 2 = 30 Ньютон

    м B движется вверх:

    T B – w B = m B a

    T B – 20 = (2) (5)

    T B – 20 = 10

    T B = 10 + 20

    T 1 = 30 Ньютон

    Сила натяжения (Т) = 30 Ньютон.

    7. Масса объекта A = 5 кг, ускорение свободного падения (g) = 10 м с -2 . Объект А движется вниз на 2,5 м / с -2 . Какая масса у B?

    Известный:

    Масса A (м A ) = 5 кг

    Ускорение свободного падения (g) = 10 м / с 2

    Ускорение объекта A (a) = 2,5 м / с 2

    Вес A (w A ) = (m A ) (g) = (5) (10) = 50 Ньютон

    Разыскивается: Масса объекта B (м B )

    Решение:

    Блок A перемещается вниз, поэтому вес объекта A (w A ) больше веса объекта B (w B ).

    Примените второй закон Ньютона:

    ΣF = m a

    w A – w B = (m A + m B ) a

    50 – (m B ) (10) = (5 + m B ) (2,5)

    50-10 м B = 12,5 + 2,5 м B

    50 – 12,5 = 2,5 м B + 10 м B

    37,5 = 12,5 м B

    м B = 3 кг

    8. Ускорение свободного падения 10 м / с 2 .Что такое сила натяжения.

    Известный:

    Масса объекта 1 (м 1 ) = 2 кг

    Масса объекта 2 (м 2 ) = 3 кг

    Ускорение свободного падения (g) = 10 м / с 2

    Вес 1 (w 1 ) = (m 1 ) (g) = (2 кг) (10 м / с 2 ) = 20 кг м / с 2

    Вес 2 (w 2 ) = (m 2 ) (g) = (3 кг) (10 м / с 2 ) = 30 кг м / с 2

    Требуется: усилие натяжения (Т)

    Решение:

    w 2 > w 1 , так что m 2 перемещается вниз, а m 1 перемещается вверх.

    Второй закон движения Ньютона:

    ΣF = m a

    w 2 – w 1 = (m 1 + m 2 ) a

    30-20 = (2 + 3) а

    10 = (5)

    a = 10/5 = 2 м / с 2 .

    Сила натяжения?

    м 2 движется вниз

    w 2 – T 2 = m 2 a

    30 – Т 2 = (3) (2)

    30 – Т 2 = 6

    Т 2 = 30 – 6

    T 2 = 24 Ньютона

    м 1 движется вверх

    T 1 – w 1 = m 1 a

    Т 1 -20 = (2) (2)

    Т 1 – 20 = 4

    Т 1 = 20 + 4

    T 1 = 24 Ньютона

    Сила натяжения (Т) = 24 Ньютона.

    Набор решений – законы Ньютона

    1.

    Мы снова используем принцип суперпозиции, чтобы решить эту проблему. проблема путем разделения проблемы на силы, действующие вдоль по горизонтали или оси X и по вертикали или ось Y.Тогда для каждой оси все силы будут направлены вдоль линия. Приписываем силам положительный или отрицательный знак, в зависимости от их направления вдоль оси, а затем добавить их алгебраически. В направлении Y нет ускорения, a y = 0. (F net ) y = ma y = m (0) = 0

    (a) и (b),
    F N – F g = 0 или F N = F г = мг.
    Для m = 2,0 кг, F N = F г = (2,0 кг) (10 м / с 2 ) = 20 N.

    (F net ) x = ma x
    F = ma x или a x = Ф / м

    Для (c) a x = 12 Н / 2,0 кг = 6,0 м / с 2 .

    Для (d) a x = 24 Н / 2,0 кг = 12,0 м / с 2 . Ускорение прямо пропорционально чистой силе.Когда вы удваиваете чистую силу, вы удваиваете ускорение.

    Для (e) a x = 12 Н / 1,0 кг = 12,0 м / с 2 . Ускорение обратно пропорционально массе объект. Когда вы уменьшаете массу вдвое, вы удваиваете ускорение.

    2.

    Теперь чистая сила в направлении X зависит от приложенного сила справа и сила трения слева.

    (F net ) x = ma x
    F – f = ma x

    1. 12 Н – 4 Н = 2,0 кг a x
      4,0 м / с 2 = a x
    2. 24 Н – 4 Н = 2,0 кг x
      10 м / с 2 = x
    В то время как прилагаемая сила F удваивается, чистая сила равна не удваивается и ускорение не удваивается. Секунда чистая сила = 20/8 x первая чистая сила = 5/2 первая равнодействующая сила. Второе ускорение = 5/2 первого ускорения = 5/2 x 4,0 м / с 2 = 10 м / с 2 .

    3.


    На Рис. 2a выше я выделил (показано пунктирной рамкой) всю систему, чтобы найти ускорение двух блоков.На рис. 2b выше я изолировал A и B отдельно, чтобы найти F AB и F BA . F AB сила B на A и F BA сила A на B. F AB – внешняя сила для системы который состоит только из A и F BA – внешняя сила к системе, состоящей только из B. F AB и F BA – внутренние силы для системы на фиг.2а. Я не использую ось Y на рисунках, потому что я не интересует нормальные силы и нет трения в проблеме. Для всей системы на рис. 2а я знаю внешняя сила, которая действует на него по оси X, и я знаю масса системы, чтобы я мог найти ее ускорение. Так как я меня интересует только направление X, я опускаю индекс x на F net и a.

    (a) F чистый внешний = (масса системы) a
    12 N = (2 + 4) кг (а) или
    12 Н / 6 кг = 2 м / с 2 = a

    Теперь мы изолируем блок A, чтобы найти F AB , поскольку мы знаем m A и a, а затем изолировать блок B, потому что мы знаем все, кроме F BA , как показано на рис. 2b. F находится в контакте только с A, а A толкает B вправо, в то время как B толкает A влево.

    Для блок А Для блок Б
    б (F net external ) вкл. A = (м A ) a (в) (F net external ) вкл. B = (м B ) a
    F – F AB = (2 кг) (2 м / с 2 ) F BA = (4 кг) (2 м / с 2 )
    12 N – F AB = 4 N F BA = 8 N
    или 12 Н – 4 Н = 8 Н = F AB

    F AB и F BA – это третий закон Ньютона движущих сил. Они равны по величине, но противоположному направлению. Ф. AB , сила B, действующая на A, имеет величину 8 Н и находится слева. F BA , сила A на B, имеет величину 8 Н и находится справа.

    4.

    Немного сложно решить (а) эту проблему без украдкой взгляните на рисунок для части (b).Есть много сил здесь, но Рис. 3-4a ниже показывает только внешних сила в горизонтальном направлении, действующая на систему два блока. Снова F net external = ma, где a – ускорение блоков, m – масса систему мы изолировали. Имеется внутренних сил действующих в системе, например, сила троса на A, F AR , сила троса на B, F BR . И мы не можем забудьте о силе A на веревке, F RA или усилие Б на канате, F RB . Я не рисовал векторов гравитационных сил на блоках или нормальное усилие поверхности на блоки, потому что они были не запрашивается и нет силы трения.

    На рисунках 3-4b и c выше показаны силы F AR , F BR, F RA и F RB , они являются внутренними по отношению к системе, показанной на рис.3-4а выше. Единственное внешнее усилие для всей системы в горизонтальном направлении F = 12 N.

    1. Таким образом, для системы, показанной на Рис. 3-4a выше,
      (F net external ) x = (масса системы) a
      12 Н = (2 + 4) кг (a) или
      12 Н / 6 кг = 2 м / с 2 = a
      Что касается части (b), давайте посмотрим на рис. 3-4b и c, которые я воспроизвожу. ниже:


      Давайте сначала разберемся с безмассовой веревкой:

      (F сетка внешняя ) на тросе = (м R ) a
      Принимая право быть позитивным,
      F RB – F RA = (0) a, где a = ускорение троса
      Так,
      F RB = F RA

      Важно отметить, что F RB и F RA , при равном и в противоположных направлениях, являются , а не Ньютоновских сил третьего закона движения, потому что они действуют на ТО ЖЕ объект.Они равны и в в противоположном направлении, потому что они должны равняться нулю, потому что безмассовый объект не имеет действующей на него чистой силы. Затем по По третьему закону Ньютона мы знаем, что:
      F RA = – F AR и F РБ = – F BR .
      Таким образом, поскольку величины этих сил равны, F RA = F RB , F AR = F BR .Мы бросаем все эти осторожные обозначения на ветер и называем F AR , F BR , F RA и F RB напряжение T. С этой информацией мы перерисовываем Рис. 3-4b ниже:

    2. Теперь мы можем выбрать любой объект, чтобы найти натяжение T в веревке. Вы можете проверить ответ, изолировав другой блок и нахождение Т.
      Для блока B, дюйм направление X, Для блока A, дюйм направление X,
      (F чистая внешняя ) x = (м B ) a (F чистый внешний ) x = (м A ) a
      12 Н – T = (4 кг) (2 м / с 2 ) = 8 Н T = (2 кг) (2 м / с 2 ) = 4 Н
      Так 12 Н – 8 Н = Т = 4 Н

    5.

    Силы, действующие на объект, – это приложенная сила F и вес объекта мг. Во всех случаях F net = ма

    (a) Для постоянной скорости a = 0 и F – mg = m (0) = 0
    или F = мг = 3,0 кг (10 м / с 2 ) = 30 Н

    (б) Для постоянного ускорения a = 3,0 м / с 2 , F – 30 Н = 3.0 кг (3 м / с 2 )
    или F = 9.0 N + 30 N = 39 N.

    6.

    1. Для всей системы (рис. 4a выше),

      F net = Ma
      F – 3 мг = (3 м) a
      9.0 Н – (0,30 кг) (10 м / с 2 ) = (0,30 кг) a
      9,0 Н – 3,0 Н = (0,30 кг) a
      20 м / с 2 = a

    2. Сначала найти натяжение в верхней части троса T (b) используйте нижний рисунок на рис. 4b выше.

      F net = Ma
      T (b) – 2 мг = (2 м) a
      T (b) – 2,0 N = (0,20 кг) (20 м / с 2 )
      T (b) – 2,0 N = 4,0 №
      T (b) = 6,0 Н

      Проверьте верхний рисунок на рис. 4b выше.

      F нетто = Ma
      9.0 N – mg – T (b) = ma
      9.0 N -1.0 N – T (b) = (0.10 кг) (20 м / с 2 ) = 2.0 N
      6.0 N = T (b)

    3. Масса 1/5 веревки = м / 5 = 0,02 кг и ее масса = мг / 5 = 0,2 н. Используйте верхний рисунок на рис. 4c выше.

      F нетто = Ma
      9,0 N – 1,2 мг – T (c) = (0,12 кг) (20 м / с 2 ) = 2,4 Н
      T (c) = 5,4 Н

      Используя нижний рисунок на рис. 4c выше,

      T (c) – 1,8 мг = T (c) – 1.8 Н = (0,18 кг) (20 м / с 2 ) = 3,6 Н
      T (c) = 5,4 Н

    7.


    1. (F net ) x = ma x (F net ) y = ma y
      26 Н – f = 2.0 кг ( x ) F N – мг = m (0)
      F N = мг = (2,0 кг) (10 м / с 2 ) = 20N

    2. f = µ к F Н = 1/5 (20 Н) = 4,0 Н
      26 Н – 4,0 Н = 2,0 кг ( x )
    3. 11 м / с 2 = x

    8.


    1. (F net ) y = ma y
      F N + 26 N sin 22,6 0 – mg = 0
      F N = + 20 N – 26 N (5/13) = 20 N – 10 N = 10 N.
      f = µ k F N = (1/5) (10 N) = 2 N
    2. (F net ) x = ma x
      26 N cos 22.6 0 – f = 2,0 кг ( x )
      26 Н (12/13) – 2 Н = 2,0 кг ( x )
      24 Н – 2 Н = 2,0 кг ( x )
      11 м / с 2 = x

    9.

    (a) и (b) Ось x выбрана в направлении ускорение. Оси, силы и составляющие сил показаны на рисунке выше.

    (c) (F net ) x = ma x
    30 N sin 30 0 = 3,0 кг ( x )
    15 N = 3,0 кг ( x )
    5,0 м / с 2 = x

    10.


    1. Теперь есть сила трения вверх по плоскости.

      (F net ) y = ma y
      F N – mg cos 30 o = 0
      F N = 30 N (0,866) = 26 N
      f = µ k F N = 0,154 (26 N) = 4,0 №

    2. (F net ) x = ma x
      30 N sin 30 o – f = 3,0 кг ( x )
      15 N – 4,0 N = 3,0 кг ( x )
      x = 11/3 м / с 2

    11.



    1. Сначала изолируйте m 1 , беря ось X в направление ускорения:
      (F net ) x = м 1 a x (F net ) y = м 1 a y
      Т – м 1 г грех 16.3 o – f = m 1 a x F N – m 1 g cos16.3 o = m 1 (0)
      F N = 25 Н (24/25) = 24 Н
      f = µ k F Н = 1/6 (24 Н) = 4 N
      Т – 25 Н (7/25) – 4 N = 2. 5 кг
      Т – 11 Н = 2,5 кг a (Уравнение 1)
      Примечание: Я опустил индекс а, единственное ускорение.

      Теперь изолируйте m 2 , принимая положительное значение:

      F net = m 2 a
      m 2 g – T = m 2 a
      20 N – T = 2.0 кг a (Уравнение 2)

      Добавление уравнения. 1 и уравнение. 2:

      20 Н – 11 Н = 4,5 кг или а = 2,0 м / с 2

    2. Подставляем a = 2,0 м / с 2 в уравнение. 2:
      20 Н – T = 2,0 кг (2,0 м / с 2 ) и Т = 16 Н.

    12.

    Рисунок для # 12a выше представляет собой эскиз объекта, движущегося по стол и все силы, действующие на него. Как объект запускается чтобы ускользнуть от круга, сила трения действует в центр круга для создания центростремительного ускорения.Я выбрал ось X вправо, потому что в данный момент на картинке центростремительное ускорение находится в центре круга или вправо. Сила трения – единственная силу в круг или, как я нарисовал, по положительному Ось X. Это трехмерный чертеж, показывающий, что для движение по часовой стрелке, в этот момент скорость вдоль положительная ось Y.Вес предмета всегда под рукой отрицательная ось Z и нормальная сила стола вдоль положительная ось Z. Теперь наша ось интересов – ось X и ось Z. Ускорение в положительном направлении оси X – центростремительное ускорение = v 2 / r.

    1. (F net external ) x = ma x
      F трение = mv 2 / r = 2 кг (2 м / с) 2 / (0.5 м) = 16 Н
    2. Нет ускорения в Z-направлении.
      (F net external ) z = ma z.
      F N – мг = m (0) или F Н = mg = (2 кг) (10 м / с 2 ) = 20 Н

      Как показано на рис. Для # 12b выше,
      Сила стола на объекте = {(F трение ) 2 + Ф. N 2 ) 1/2
      = {(16) 2 + (20) 2 } 1/2
      = {656} 1/2 N = 25.6 N
      tan Q = F N / F трение = 20/16 = 1,25
      Q = 51,3 o

    13.


    1. Центростремительное ускорение a = v 2 / r = (3 м / с) 2 /0,5 м = 18 м / с 2 .Объект движется против часовой стрелки. направление, чтобы направления скоростей были такими, как показано на рис. 7а выше. Помните, что скорость всегда касательная к пути. Таким образом,
    2. в точке B скорость вправо,
    3. в точке T скорость слева, а
    4. в точке S скорость снижена.
    5. В точке B ускорение находится в центре круга или выше.В точке B мы поднимаем ось X вверх.
    6. В точке T ускорение по кругу или вниз. В T ось X направлена ​​вниз.
    7. В точке S ускорение направо, а ось X Направо. Все ускорения в направлении чистой силы.

      Для всех случаев F нетто = ma = 2 кг (18 м / с 2 ) = 36 Н.

      Глядя на рис.7b ниже мы видим, что в точках B и T там не являются компонентами силы в любом направлении, кроме Направление X, поэтому мы опускаем индексы на F net .

    8. На B,

      F net = ma
      F или на B – mg = ma
      F или на B – 20 N = 36 N
      F или на B = 56 N

    9. В T я нарисовал F или в T вниз, но я должен использовать уравнение для окончательного решения, поэтому я пишу его с ± найти его знак и, следовательно, его направление.

      F net = ma
      ± F или при T + 20 N = 36 N, поэтому
      F или при T = +16 N, или он не работает.

    10. На S, F или на S должны иметь компонент в центр или вдоль оси + X, чтобы произвести центростремительное ускорение и составляющая по оси + Y Таким образом, результирующая сила по оси Y равна нулю, сохраняя величину скорости для постоянной равномерного кругового движения. Таким образом, мы должны смотреть на F net по осям X и Y.

      (F net ) x при S = ma x (F net ) л при S = ma y
      (F или ) x при S = 36 Н (ф. или ) л. при S – mg = m (0)
      (F или ) y при S = mg = 20 N
      F или при S = {(F или ) 2 x при S + (F или ) 2 905 10 x при S } 1/2
      F или при S = {(36) 2 + (20) 2 } 1/2 N = 41 N
      желто-коричневый Q = (F или ) y / (F или ) x = 20 Н / 36 Н = 0. 55.
      Q = 29 o

    14.

    1. Силы, показанные на рис.8 ниже:

    2. (F net ) x = ma x (F net ) y = ma y
      -mg sin Q = ma x T – мг cos Q = mv 2 / L

      Ускорение в направлении X = -g sin Q
      Ускорение в направлении Y = v 2 / L, где v – скорость объекта в этот момент, а L – радиус дуги окружности.

      a y = v 2 / L = T / m – g cos Q

    3. Как правило, x = -g sin Q
      (i) при максимальном смещении, x = -g sin Q max
      (ii) в положении равновесия, Q = 0, sin Q = 0, a x = 0.
      Как правило, T- mg cos Q = mv 2 / L.
      1. При максимальном смещении боб моментально подходит в покое,
        v = 0 и T = mg cos Q max .
      2. В положении равновесия v = v max ,
        T = mg cos 0 0 + mv max 2 / л = мг + мв макс 2 / л.

    15.


    1. Силы, действующие на объект, – это его вес м г и натяжение Т в тетиве.Ускорение находится в центре или, в этот момент, вправо. Мы возьмите ось X в направлении ускорения и ось Y перпендикулярна ему. Вес м г составляет в отрицательном направлении Y, но T ни в одном в направлении X и Y, поэтому мы берем компоненты, как показано на рисунке 9 выше. Ускорение в направлении X – центростремительное ускорение v 2 / r.Там есть нет ускорения в направлении Y.


    2. (F net ) x = ma x (F net ) y = ma y
      T sin Q = mv 2 / r (Уравнение 1) T cos Q – мг = 0 или
      T cos Q = мг (Уравнение 2)

      Деление уравнения. 1 по формуле. 2:
      T sin Q / T cos Q = tan Q = (mv 2 / r) / (mg) = v 2 / rg (Уравнение 3)
      Из геометрии видим, что r / L = sin Q или г = L sin Q (Уравнение 4)
    3. Подставляя уравнение.4 в уравнение. 3:

      tan Q = v 2 / (L sin Q) g или (загар Q) (L sin Q) g = v 2 и
      v = (Lg sin Qtan Q) 1/2

    4. v = (пройденное расстояние / время) = 2pr / T или
      T = 2pr / v = 2p (L sin Q) / v =
      T = 2p (L sin Q) / (Lg sin Qtan Q) 1/2
      = 2p (L sin Q) / (Lg sin 2 Q / cos Q) 1/2 =
      T = 2p (L cos Q / g) 1/2

    16.



    1. Для движения по горизонтальной окружности, из решения до # 15, мы видим, что:

      T sin Q = mv 2 / L sinQ или
      v 2 = TL sin 2 Q / м (1)

      Кроме того, T cos Q = мг или cos Q = mg / T

      Из рис. а видно, что:

      sin Q = (T 2 – м 2 г 2 ) 1/2 / T (2)

      Подставляя уравнение.2 в уравнение. (1):

      v 2 = TL / м (T 2 – m 2 г 2 ) / T 2
      = TL / м {1 – (мг / т) 2 } или
      v = (TL / м) 1/2 {1 – (мг / т) 2 } 1/2

    2. Для вертикального круга максимальное натяжение возникает, когда объект находится в самой низкой точке, а натяжение и вес объекта находятся в противоположных направлениях.В этом случае
      T – мг = mv 2 / л или v = {(T – мг) л / м} 1/2

    17.

    1. Графики положения по осям x и y как функции времени прямые.Наклоны этих линий равны постоянные скорости, v x и v y , соответственно. При постоянной скорости ускорение отсутствует. ни силы. F x = 0. F y = 0.
    2. Для снаряда объект перемещается по горизонтали расстояния одновременно с постоянной скоростью. F x = 0.Идет постоянный разгон вниз. F г = – постоянная.
    3. Скорость частицы постоянна. Скорость всегда касается окружности. Это равномерное круговое движение с ускорением к центру. Это постоянно по величине, но меняет направление.

      F x = – (постоянный) x и F y = – (постоянная) y.
      F = F x i + F y j = – (константа) (x i + y j ) или F = – (константа) r , где r – радиальный вектор.

    4. Наклон v x по сравнению с t = k = a x .
      Наклон v y по сравнению с t = -k ’= a y , где k и k ’- константы.
      F x = c и F y = -c ’, где c и c’ – новые константы.

    18.

    (б)

    (a) Сначала изолируйте всю систему, как изображенный на рис.11а выше. Для этой системы T 2 , Т 3 , усилие F МП платформы на человека, и сила F pm человека на Платформа – это внутренние силы. Внешние силы – Т 1 , вес человека (мг) человек , а вес платформы (мг) платформа .

    F Чистая внешняя = Ma
    T 1 -1000 Н – 600 Н = (100 + 60) кг (5.0 м / с 2 )
    T 1 = 2400 Н
    T 2 = T 3 = T 1 /2 = 1200 Н

    (c) Теперь изолируйте человека, как показано на Рис. 11b выше:

    F net external = m man a
    T 3 – (mg) man + F mp = 100 кг (5 м / с 2 )
    1200 Н – 1000 Н + F mp = 500 N

    Сила платформы на человека F mp = 300 Н.

    В качестве проверки изолируем платформу, как на рис. 11c выше:

    F net external = m platform a
    T 2 – (mg) platform – F pm = 60 кг (5 м / с 2 )
    1200 Н – 600 Н – F mp = 300 Н

    Сила человека на платформе F pm = 300 Н. F mp и F pm – третий закон Ньютона. движущих сил.Они равны по величине и противоположны по направлению.

    19.

    Сначала изолируйте всю систему (рис. 12a выше). На весь системы, внешние силы – это приложенная сила F, фрикционная сила f 2 поверхности на 2 м, нормальная сила F Н на 1 и 2 поверхности на двух блоках и притяжение земли к предметам или весу объектов, (м 1 + м 2 ) г.

    При перемещении объектов вправо действует f 2 Слева.

    (F net ) x = Ma x (F net ) y = Ma y
    45 Н – ф 2 = 6,0 кг F N на 1 и 2 – 60 N = 0
    F N на 1 и 2 = 60 N
    f 2 = µ k F N на 1 и 2 = 0.25 (60 Н) = 15 Н
    45 Н – 15 Н = 6,0 кг
    5,0 м / с 2 =

    Теперь изолируйте m 1 (рис. 12b выше). Должна быть сила трения f 1 на м 1 из-за м 2 по Право производить ускорение м 1 .

    (F net ) x = м 1 a x (F net ) y = м 1 a y
    f 1 = 2,0 кг (5 м / с 2 ) F N1 – 20 Н = 0
    F N1 = 20 Н и f 1 = µ с (20 Н)
    µ с (20 N) = 10 Н
    µ с = 1/2

    В качестве контрольного изолятора m 2 (Инжир.12c выше). По третьему закону Ньютона m 1 должно приложить силу f 1 на м 2 Слева.

    (F net ) x = m 2 a x
    45 N – f 2 – f 1 = 4,0 кг (5 м / с 2 ) или
    45 Н – 15 Н – f 1 = 20 Н

    и снова,

    f 1 = 10 Н с µ с (20 Н) = 10 Н и
    µ с = 1/2.

    20.

    Когда блок начинает скользить вниз, сила трения f противодействует движение.

    (F сетка ) x = ma x (F net ) y = ma y
    N = mv 2 / R f – мг = 0 или f = mg = µ с N
    Начиная с v = 2pR / T o = 2pRf o N = мг / µ с (Уравнение 1)
    N = m (4p 2 R 2 f o 2 ) / R (Уравнение 2)

    Подставляя уравнение.1 в уравнение. 2 дает: f o = (1 / 2p) (г / µ с R) 1/2

    21.


    Возьмите ось X в направлении ускорения или вверх плоскости и перпендикулярной ей оси Y
    (рис. 14 выше).

    (F net ) x ma x
    F cos 37 o – mg sin 37 o – f = ma
    60 N (4/5) – 20 N (3/5) – f = 2.0 кг (а)
    48 Н – 12 Н – f = 0 (Уравнение 1)

    (F net ) y = ma y
    F N – F sin 37 o – mg cos 37 o = m (0)
    F N = 60 N (3/5) + 20 N (4/5) = 52 N
    f = µ k F N = 1/2 (52 N) = 26 Н (Уравнение 2)

    Подставляя уравнение.2 в уравнение. 1:

    48 Н – 12 Н – 26 Н = 2,0 кг (а) или а = 5,0 м / с 2

    22.

    1. Сила полезная на салазках = м салазки a салазки = 10 кг (2,0 м / с 2 ) = 20 Н = сила человека на салазках.
    2. Согласно третьему закону Ньютона, сила человека на санях = сила санки на человека
      = 20 Н.
    3. Сила полезной нагрузки на человека = м человек a человек = 60 кг (2,0 м / с 2 ) = 120 Н = сила трения снега на человека – сила саней на человека. 120 Н = сила трения снега на человека – 20 Н. Сила трения снега на человеке = 140 Н.

    23.



    1. Пусть F os = сила шкалы на объекте когда лифт движется с постоянной скоростью. Для постоянная скорость, F net = ma = m (0).
      F os – mg = 0 и F os = mg = (10 кг) (10 м / с 2 ) = 100 Н.К третьему Ньютону закон, сила шкалы на объекте = сила объект на шкале. Показание шкалы – 100. №
    2. Пусть F ‘ os = сила шкалы на объекте когда лифт движется вверх с постоянным ускорением 5 м / с 2 .
      F net = ma = (10 кг) (5 м / с 2 ).
      F ‘ os – мг = 50 Н или F’ os = 50 Н + 100 Н = 150 Н.
      F ‘ os = F’ , поэтому = показание шкалы = 150 №

    24.

    F net = mg – f = ma или a = г – ж / м. Свинцовый шар имеет большую плотность, чем деревянный. сфера. Объем сфер = 4/3 пр 3 а масса m сфер = плотность x объем.Свинцовая сфера имеет тот же объем, что и деревянная сфера, так как их радиусы равны идентично, но имеет большую массу, потому что имеет большую плотность. Поскольку a = g – f / m, сфера большей массы имеет большее ускорение.

    Второй закон Ньютона – AP Physics 1

    Если вы считаете, что контент, доступный через Веб-сайт (как определено в наших Условиях обслуживания), нарушает или другие ваши авторские права, сообщите нам, отправив письменное уведомление («Уведомление о нарушении»), содержащее в информацию, описанную ниже, назначенному ниже агенту.Если репетиторы университета предпримут действия в ответ на ан Уведомление о нарушении, оно предпримет добросовестную попытку связаться со стороной, которая предоставила такой контент средствами самого последнего адреса электронной почты, если таковой имеется, предоставленного такой стороной Varsity Tutors.

    Ваше Уведомление о нарушении прав может быть отправлено стороне, предоставившей доступ к контенту, или третьим лицам, таким как в качестве ChillingEffects.org.

    Обратите внимание, что вы будете нести ответственность за ущерб (включая расходы и гонорары адвокатам), если вы существенно искажать информацию о том, что продукт или действие нарушает ваши авторские права.Таким образом, если вы не уверены, что контент находится на Веб-сайте или по ссылке с него нарушает ваши авторские права, вам следует сначала обратиться к юристу.

    Чтобы отправить уведомление, выполните следующие действия:

    Вы должны включить следующее:

    Физическая или электронная подпись правообладателя или лица, уполномоченного действовать от их имени; Идентификация авторских прав, которые, как утверждается, были нарушены; Описание характера и точного местонахождения контента, который, по вашему мнению, нарушает ваши авторские права, в \ достаточно подробностей, чтобы позволить репетиторам университетских школ найти и точно идентифицировать этот контент; например нам требуется а ссылка на конкретный вопрос (а не только на название вопроса), который содержит содержание и описание к какой конкретной части вопроса – изображению, ссылке, тексту и т. д. – относится ваша жалоба; Ваше имя, адрес, номер телефона и адрес электронной почты; а также Ваше заявление: (а) вы добросовестно полагаете, что использование контента, который, по вашему мнению, нарушает ваши авторские права не разрешены законом, владельцем авторских прав или его агентом; (б) что все информация, содержащаяся в вашем Уведомлении о нарушении, является точной, и (c) под страхом наказания за лжесвидетельство, что вы либо владелец авторских прав, либо лицо, уполномоченное действовать от их имени.

    Отправьте жалобу нашему уполномоченному агенту по адресу:

    Чарльз Кон Varsity Tutors LLC
    101 S. Hanley Rd, Suite 300
    St. Louis, MO 63105

    Или заполните форму ниже:

    MS-PS2-1 Движение и устойчивость: силы и взаимодействия

    Учащиеся, демонстрирующие понимание, могут:

    МС-ПС2-1. [Уточняющее заявление: Примеры практических проблем могут включать в себя воздействие столкновений между двумя автомобилями, между автомобилем и неподвижными объектами, а также между метеором и космическим аппаратом.] [ Граница оценки: оценка ограничивается вертикальными или горизонтальными взаимодействиями в одном измерение. ]
    Приведенные выше ожидаемые результаты были разработаны с использованием следующих элементов из документа NRC A Framework for K-12 Science Education :

    Наука и инженерная практика

    Построение объяснений и разработка решений

    Построение объяснений и разработка решений в 6–8 основано на опыте K – 5 и включает в себя построение объяснений и разработку решений, подкрепленных многочисленными источниками доказательств, согласующихся с научными идеями, принципами и теориями.

    • Применяйте научные идеи или принципы для разработки объекта, инструмента, процесса или системы.

    Основные дисциплинарные идеи

    PS2.A: Силы и движение

    Сквозные концепции

    Системы и системные модели

    – – – – – – – – – – – – – – – – – – – – – – – – – – – – – – – – – –

    Connections to Engineering, Technology,

    & nbsp and Applications of Science

    Влияние науки, техники и технологий на общество и мир природы

    • Использование технологий и любые ограничения на их использование обусловлены индивидуальными или общественными потребностями, желаниями и ценностями; по результатам научных исследований; а также различиями в таких факторах, как климат, природные ресурсы и экономические условия.

    Подключения к другим DCI в этом классе:

    MS.PS3.C

    Составление DCI по классам:

    3.PS2.A ; HS.PS2.A

    Стандартные соединения основных государственных стандартов:

    ELA / Грамотность –
    РСТ.6-8.1 Приводите конкретные текстовые свидетельства для поддержки анализа научных и технических текстов, уделяя внимание точным деталям объяснений или описаний. (MS-PS2-1)
    RST.6-8.3 Точно соблюдайте многоступенчатую процедуру при проведении экспериментов, измерений или технических задач. (MS-PS2-1)
    WHST.6-8.7 Выполните короткие исследовательские проекты, чтобы ответить на вопрос (включая вопрос, созданный самостоятельно), опираясь на несколько источников и создавая дополнительные связанные, сфокусированные вопросы, которые позволяют для нескольких направлений исследования. (MS-PS2-1)
    Математика –
    МП.2 Размышляйте абстрактно и количественно. (MS-PS2-1)
    6.NS.C.5 Поймите, что положительные и отрицательные числа используются вместе для описания величин, имеющих противоположные направления или значения; Используйте положительные и отрицательные числа для представления количеств в реальном контексте, объясняя значение 0 в каждой ситуации. (MS-PS2-1)
    6.EE.A.2 Записывайте, читайте и оценивайте выражения, в которых буквы заменяют числа. (MS-PS2-1)
    7.EE.B.3 Решайте многоступенчатые реальные и математические задачи, поставленные с положительными и отрицательными рациональными числами в любой форме, используя инструменты стратегически. Применять свойства операций для вычисления с числами в любой форме; конвертировать между формами по мере необходимости; и оценить разумность ответов с помощью мысленных вычислений и стратегий оценки. (MS-PS2-1)
    7.EE.B.4 Используйте переменные для представления величин в реальных или математических задачах и создавайте простые уравнения и неравенства для решения проблем, рассуждая о величинах . (MS-PS2-1)

    Учащиеся, демонстрирующие понимание, могут:

    МС-ПС2-1. [Уточняющее заявление: Примеры практических проблем могут включать в себя столкновение двух автомобилей, между автомобилем и неподвижными объектами, а также между метеором и космическим аппаратом.] [ Граница оценки: оценка ограничивается вертикальными или горизонтальными взаимодействиями в одном измерении. ]
    Приведенные выше ожидаемые результаты были разработаны с использованием следующих элементов из документа NRC A Framework for K-12 Science Education :

    Наука и инженерная практика

    Построение объяснений и разработка решений

    Построение объяснений и разработка решений в 6–8 основано на опыте K – 5 и включает в себя построение объяснений и разработку решений, подкрепленных многочисленными источниками доказательств, согласующихся с научными идеями, принципами и теориями.

    • Применяйте научные идеи или принципы для разработки объекта, инструмента, процесса или системы.

    Основные дисциплинарные идеи

    PS2.A: Силы и движение

    Сквозные концепции

    Системы и системные модели

    – – – – – – – – – – – – – – – – – – – – – – – – – – – – – – – – – –

    Connections to Engineering, Technology,

    & nbsp and Applications of Science

    Влияние науки, техники и технологий на общество и мир природы

    • Использование технологий и любые ограничения на их использование обусловлены индивидуальными или общественными потребностями, желаниями и ценностями; по результатам научных исследований; а также различиями в таких факторах, как климат, природные ресурсы и экономические условия.

    Подключения к другим DCI в этом классе:

    MS.PS3.C

    Составление DCI по классам:

    3.PS2.A ; HS.PS2.A

    Стандартные соединения основных государственных стандартов:

    ELA / Грамотность –
    РСТ.6-8.1 Приводите конкретные текстовые свидетельства для поддержки анализа научных и технических текстов, уделяя внимание точным деталям объяснений или описаний. (MS-PS2-1)
    RST.6-8.3 Точно соблюдайте многоступенчатую процедуру при проведении экспериментов, измерений или технических задач. (MS-PS2-1)
    WHST.6-8.7 Выполните короткие исследовательские проекты, чтобы ответить на вопрос (включая вопрос, созданный самостоятельно), опираясь на несколько источников и создавая дополнительные связанные, сфокусированные вопросы, которые позволяют для нескольких направлений исследования. (MS-PS2-1)
    Математика –
    МП.2 Размышляйте абстрактно и количественно. (MS-PS2-1)
    6.NS.C.5 Поймите, что положительные и отрицательные числа используются вместе для описания величин, имеющих противоположные направления или значения; Используйте положительные и отрицательные числа для представления количеств в реальном контексте, объясняя значение 0 в каждой ситуации. (MS-PS2-1)
    6.EE.A.2 Записывайте, читайте и оценивайте выражения, в которых буквы заменяют числа. (MS-PS2-1)
    7.EE.B.3 Решайте многоступенчатые реальные и математические задачи, поставленные с положительными и отрицательными рациональными числами в любой форме, используя инструменты стратегически. Применять свойства операций для вычисления с числами в любой форме; конвертировать между формами по мере необходимости; и оценить разумность ответов с помощью мысленных вычислений и стратегий оценки. (MS-PS2-1)
    7.EE.B.4 Используйте переменные для представления величин в реальных или математических задачах и создавайте простые уравнения и неравенства для решения проблем, рассуждая о величинах . (MS-PS2-1)

    Учащиеся, демонстрирующие понимание, могут:

    МС-ПС2-1. [Уточняющее заявление: Примеры практических проблем могут включать в себя столкновение двух автомобилей, между автомобилем и неподвижными объектами, а также между метеором и космическим аппаратом.] [ Граница оценки: оценка ограничивается вертикальными или горизонтальными взаимодействиями в одном измерении. ]
    Приведенные выше ожидаемые результаты были разработаны с использованием следующих элементов из документа NRC A Framework for K-12 Science Education :

    Наука и инженерная практика

    Построение объяснений и разработка решений

    Построение объяснений и разработка решений в 6–8 основано на опыте K – 5 и включает в себя построение объяснений и разработку решений, подкрепленных многочисленными источниками доказательств, согласующихся с научными идеями, принципами и теориями.

    • Применяйте научные идеи или принципы для разработки объекта, инструмента, процесса или системы.

    Основные дисциплинарные идеи

    PS2.A: Силы и движение

    Сквозные концепции

    Системы и системные модели

    – – – – – – – – – – – – – – – – – – – – – – – – – – – – – – – – – –

    Connections to Engineering, Technology,

    & nbsp and Applications of Science

    Влияние науки, техники и технологий на общество и мир природы

    • Использование технологий и любые ограничения на их использование обусловлены индивидуальными или общественными потребностями, желаниями и ценностями; по результатам научных исследований; а также различиями в таких факторах, как климат, природные ресурсы и экономические условия.

    Подключения к другим DCI в этом классе:

    MS.PS3.C

    Составление DCI по классам:

    3.PS2.A ; HS.PS2.A

    Стандартные соединения основных государственных стандартов:

    ELA / Грамотность –
    РСТ.6-8.1 Приводите конкретные текстовые свидетельства для поддержки анализа научных и технических текстов, уделяя внимание точным деталям объяснений или описаний. (MS-PS2-1)
    RST.6-8.3 Точно соблюдайте многоступенчатую процедуру при проведении экспериментов, измерений или технических задач. (MS-PS2-1)
    WHST.6-8.7 Выполните короткие исследовательские проекты, чтобы ответить на вопрос (включая вопрос, созданный самостоятельно), опираясь на несколько источников и создавая дополнительные связанные, сфокусированные вопросы, которые позволяют для нескольких направлений исследования. (MS-PS2-1)
    Математика –
    МП.2 Размышляйте абстрактно и количественно. (MS-PS2-1)
    6.NS.C.5 Поймите, что положительные и отрицательные числа используются вместе для описания величин, имеющих противоположные направления или значения; Используйте положительные и отрицательные числа для представления количеств в реальном контексте, объясняя значение 0 в каждой ситуации. (MS-PS2-1)
    6.EE.A.2 Записывайте, читайте и оценивайте выражения, в которых буквы заменяют числа. (MS-PS2-1)
    7.EE.B.3 Решайте многоступенчатые реальные и математические задачи, поставленные с положительными и отрицательными рациональными числами в любой форме, используя инструменты стратегически. Применять свойства операций для вычисления с числами в любой форме; конвертировать между формами по мере необходимости; и оценить разумность ответов с помощью мысленных вычислений и стратегий оценки. (MS-PS2-1)
    7.EE.B.4 Используйте переменные для представления величин в реальных или математических задачах и создавайте простые уравнения и неравенства для решения проблем, рассуждая о величинах . (MS-PS2-1)

    * Ожидаемые результаты, отмеченные звездочкой, объединяют традиционное научное содержание и инженерное дело посредством практической или дисциплинарной основной идеи.

    Оставить комментарий